Post UWSA 1.

¡Supera tus tareas y exámenes ahora con Quizwiz!

A 33-year-old woman with a 9-year history of ulcerative colitis comes to the office due to concern about her risk of colon cancer. One of her close family friends died recently from this cancer. The patient wants to "undergo all the tests for colon cancer." She has no other comorbid conditions and is under the care of a gastroenterologist who has kept her disease well controlled with sulfasalazine. Compared with sporadic colorectal carcinoma, colorectal malignancy arising in this patient would more likely exhibit which of the following features? A.Early APC gene mutation B.Low-grade histology C.Multifocal origin D.Origin from an adenomatous polyp

C Inflammatory bowel disease (IBD) is associated with a significant risk for colorectal carcinoma (CRC); up to 30% of patients with IBD develop CRC in their lifetime. Patients with ulcerative colitis, particularly pancolitis, are at highest risk. Colitis-associated colorectal cancer (CA-CRC) occurs in areas of chronic inflammation, and the risk is proportionate to the duration and severity of inflammation. (Choices A, B, and D) Unlike sporadic CRC, which typically progresses slowly from a singular dysplastic polyp, CA-CRC is typically more aggressive, often evolves from flat (nonpolypoid) lesions, and is frequently multifocal (ie, multiple synchronous carcinomas) at diagnosis. CA-CRC often affects a younger population than sporadic CRC. These malignancies are more likely to have a higher histopathologic grade, with poorly differentiated or anaplastic cells and a high number of mucinous or signet ring cells. The molecular pathogenesis of CA-CRC is also different than that of sporadic disease, with p53 mutations occurring early in the course of malignant development and APC mutations occurring much later. Given the high risk for CRC and the difficulty of colonoscopic visualization of flat or multifocal lesions, it is important to regularly monitor IBD via colonoscopy with random biopsies.

Embryonal origin for melanocyte?

Neural crest cell

A young couple has undergone a successful in vitro fertilization procedure. The father has cystic fibrosis and the mother has a sister with cystic fibrosis. The father as well as the mother's sister are both known to have ΔF508 mutations, but the mother's carrier status is unknown. Before making the decision to conceive, the couple underwent extensive genetic counseling regarding the potential risks of having a child with cystic fibrosis. The family pedigree is diagrammed below with the unborn child marked by the red arrow.

1/3

A primary care physician is invited to give a presentation to a local employer. He is a member of a large multi-specialty medical group, which he joined less than a year ago after completing residency. During the presentation, the physician discusses the various medical specialties available within the group, as well as the range of ancillary services the group offers at its facilities. In the ensuing discussion, the physician learns that the employer has been facing financial difficulties. The employer wishes to continue offering health care coverage to its employees but needs to reduce expenditures. As a result, the employer would like to negotiate a contract in which the medical group would provide care to all the company's employees in exchange for a set monthly fee per employee. Which of the following payment methods best describes this type of health care financing arrangement? A.Capitation B.Discounted fee-for-service C.Global payment D.Patient-centered medical home E.Point-of-service

A

A 34-year-old man comes to the hospital due to several hours of difficulty swallowing, dry mouth, and blurred vision. The patient has a history of major depression. Examination shows mydriasis and poorly reactive pupils. Electrodiagnostic studies reveal normal nerve conduction velocity but decreased compound muscle action potential (CMAP). Rapid, repetitive nerve stimulation leads to facilitation of CMAP. Further review of this patient's medical history would most likely reveal which of the following? A.Consumption of home-canned food B.Diarrheal disease after eating undercooked poultry C.Episodic diplopia and ptosis D.Occupational exposure to insecticides E.Recent consumption of puffer fish F.Tricyclic antidepressant overdose

A The combination of nicotinic (eg, diplopia, dysphagia) and muscarinic blockade (eg, dry mouth) is strongly suggestive of food poisoning with Clostridium botulinum toxin, a highly potent preformed neurotoxin. The toxin inhibits acetylcholine release from presynaptic nerve terminals at the neuromuscular junction (NMJ), thereby preventing muscular contraction. This effect can be seen on electromyography as a decrease in the compound muscle action potential (CMAP, or electrical response of the muscle) following stimulation of a motor nerve. High-rate, repetitive nerve stimulation improves the deficit as the rapid depolarization rate increases calcium concentration in the presynaptic nerve terminal, helping mobilize additional acetylcholine vesicles. The anaerobic environment within canned foods contaminated by C botulinum spores allows for germination and growth of the organism. The toxin (produced by vegetative bacteria) is not actively secreted, remaining intracellular until autolysis causes its release into food. The toxin is readily destroyed by heat; however, if food containing the toxin is not cooked properly, the classic diplopia, dysphagia, and dysphonia ("3 Ds") can develop within 12-36 hours of consumption.

A 67-year-old man is brought to the hospital after he developed sudden-onset altered mental status. The patient was taking care of some yardwork alongside his 17-year-old grandson during a hot summer day. The grandson does not have any symptoms. On examination, the patient's temperature is 41 C (105.8 F), blood pressure is 90/60 mm Hg, pulse is 120/min, and respirations are 24/min. Examination shows warm and dry skin. The patient is not oriented to time, place, or person. Deep tendon reflexes and muscle tone are normal. Which of the following findings in this patient most likely explains why the patient developed symptoms while his grandson remained asymptomatic? A.Decreased effective epidermal area available for heat transfer B.Decreased vasodilation of the splanchnic vasculature C.Increased heat transfer to the skin due to reduced subcutaneous fat D.Increased number of sweat glands E.Increased peripheral vasodilation of the skin

A This elderly patient developed hyperthermia and altered mental status while working outside on a hot summer day. This presentation suggests exertional heat stroke, a life-threatening multisystem disorder characterized by hyperthermia (typically >40 C [104 F]) associated with central nervous system dysfunction (eg, encephalopathy, as seen in this disoriented patient). Other classic manifestations of heat stroke include tachycardia, tachypnea, hypotension, flushing, and end-organ dysfunction (eg, pulmonary edema, renal or hepatic failure). Diaphoresis may or may not be present on examination, depending on hydration status. Normal thermoregulatory response to elevated temperatures include increased sweating and increased cardiac output. Furthermore, there is significant peripheral vasodilation and splanchnic vasoconstriction, which shunts blood to the periphery to aid with heat transfer. However, elderly patients are at particularly high risk of developing heat-related illness due to several features of normal aging that impair this response, including: Tonic contraction of the peripheral vasculature, which limits heat transfer to the skin Reduced sweat gland density, which limits the ability to dissipate heat via evaporation Loss of rete pegs and dermal capillaries, which reduces the effective epidermal area available for heat transfer In addition, many elderly patients have medical conditions (eg, congestive heart failure) or are taking medications that may limit cardiac output (eg, beta blockers), diaphoresis (eg, anticholinergics), or blood volume available for heat transfer (eg, diuretics).

A 36-year-old man is hospitalized due to worsening fever, productive cough, and shortness of breath. He has no prior medical conditions but experienced an influenza-like illness during the previous week. Temperature is 38.3 C (102 F), blood pressure is 100/50 mm Hg, pulse is 122/min, and respirations are 26/min. Pulse oxymetry is 86% on room air. On physical examination, the patient is ill-appearing and lethargic. There are crackles over the right lower lung. Leukocyte count is 20,000 cells/mm3. Chest imaging reveals extensive parenchymal consolidation and cavitations suggestive of abscesses in the right middle and lower lobes. The patient is endotracheally intubated for mechanical ventilation due to worsening respiratory failure. Culture of the tracheal aspirate grows gram-positive cocci in clusters. Which of the following bacterial virulence factors most likely contributed to this patient's necrotizing infection? A.Leukocidin B.Lipooligosaccharide C.M protein D.Polysaccharide capsule

A Young, healthy individuals with recent influenza infection are at risk for rapidly progressive, necrotizing pneumonia (eg, pulmonary cavitations/abscesses) due to infection with Staphylococcus aureus, a gram-positive cocci that grows in grapelike clusters. S aureus colonizes the nares and skin of up to 30% of individuals in the community setting. Although it typically behaves as a commensal organism, breakdowns in the cutaneous or mucosal barrier (eg, respiratory epithelial damage due to influenza) can create a portal of entry for the bacteria and lead to skin or soft-tissue infection (eg, boil) or deeper, life-threatening disease (eg, pneumonia, osteomyelitis). The most virulent strains of S aureus have been infected with a bacteriophage that transmits genes for Panton-Valentine leukocidin (PVL), a cytotoxin that destroys leukocytes and causes tissue necrosis. PVL is primarily seen in community-acquired strains of S aureus that are methicillin-resistant (methicillin-resistance is conferred by a different mobile genetic element [mecA]).

MOA of clomiphene?

Antagonist at the estrogen receptor in hypothalamus. this leads to increase in LH AND FSH- OVULATION

A 60-year-old man comes to the office due to a 4-month history of increasing midthoracic back pain, which is aggravated by coughing. The patient has also had right thigh pain but no numbness or weakness in the legs and no bowel or bladder incontinence. He has no other medical conditions. Vital signs are within normal limits. On examination, there is tenderness over the eighth and ninth thoracic vertebrae. Neurologic examination is normal. The prostate is normal in size and has no palpable nodules. Imaging studies reveal enlarged vertebral bodies with cortical thickening. Serum testing for which of the following would most likely help establish a diagnosis in this patient? A.25-hydroxyvitamin D level B.Alkaline phosphatase level C.Erythrocyte sedimentation rate D.Parathyroid hormone level E.Prostate-specific antigen level

B This older patient with multifocal bone pain has enlarged vertebral bodies with cortical thickening on imaging, which is consistent with Paget disease of bone (PD). PD is characterized by excessive and disordered bone formation. It commonly affects the skull, long bones of the extremities, and vertebral column. In addition to pain and deformity, vertebral involvement can lead to spinal stenosis, nerve compression, and compression fracture. The increased formation of new bone in PD is associated with an elevated serum alkaline phosphatase level, but because the changes occur slowly, serum calcium and phosphorus levels are often normal. Radiographs typically reveal lytic or mixed lytic-sclerotic lesions, thickening of cortical and trabecular bone, and bony deformities. In the vertebrae, bony enlargement and cortical thickening may create an appearance resembling a picture frame. (Choice A) The level of 25-hydroxyvitamin D reflects total body vitamin D stores and is useful in the diagnosis of osteomalacia. In addition to bone pain, osteomalacia causes muscle weakness and difficulty walking. X-ray reveals diffuse demineralization, often with insufficiency fractures, rather than cortical thickening.

A 45-year-old man with a history of chronic alcohol use disorder is brought to the emergency room due to altered mental status. The patient appears malnourished. He is given thiamine, folic acid, a multivitamin, and dextrose-containing intravenous fluids. However, the patient develops marked muscle weakness a few hours later. Laboratory studies reveal a serum phosphate concentration of 0.5 mg/dL (normal: 2.5-4.5). Which of the following is the most likely cause of this patient's low serum phosphate level? A.Decreased renal proximal tubular reabsorption B.Increased colonic excretion of phosphate C.Increased extracellular binding with calcium D.Increased uptake by bone cells E.Redistribution of phosphate into hepatic and muscle cells

E In malnourished individuals giving carbs aka dextrose causes insulin to increase which pushes phosphate into the cell. Phosphorus is involved in multiple biologic processes, including cellular energy metabolism, bone formation, and acid-base homeostasis. Although biologically active phosphorus is largely found intracellularly, serum phosphorus levels are often reflective of available body stores and are maintained through the action of hormones (eg, parathyroid hormone, calcitriol, FGF-23) on the small intestines, bones, and kidneys. Malnourishment (eg, due to chronic alcohol use disorder) results in the depletion of phosphate, although serum levels may remain normal due to transcellular shifts. Reintroduction of carbohydrates (ie, dextrose-containing intravenous fluids) increases insulin secretion, which stimulates the redistribution of phosphate from the serum into muscle and hepatic cells for use during glycolysis (eg, formation of ATP, 2-3 diphosphoglycerate). This leads to profound hypophosphatemia; lack of adequate intracellular phosphate can result in failure of cellular energy metabolism, producing the clinical features of refeeding syndrome (eg, muscular weakness, arrhythmias, congestive heart failure).

A 23-year-old man comes to the physician complaining of right-sided testicular swelling. He first noticed the swelling 1 week ago while getting ready for work. He denies any testicular pain or history of trauma. However, he has noticed a heavy, pressing sensation involving his scrotum and lower abdomen. Physical examination reveals asymmetric swelling of the right testis, and subsequent ultrasonography shows a solid testicular mass. If malignant, this patient's tumor is most likely to spread to which of the following lymph node groups? A.Superficial inguinal B.Deep inguinal C.External iliac D.Common iliac E.Inferior mesenteric F.Para-aortic

F recall that testis used to intraabdominal organ. During fetal development, the testes originate within the retroperitoneum and establish their arterial supply from the abdominal aorta. The testes subsequently descend through the inguinal canals into the scrotum, taking with them their arterial, venous, and lymphatic supplies. Thus, lymph from the testes drains through lymph channels directly back to the para-aortic (retroperitoneal) lymph node

A 34-year-old man is admitted to the hospital with acute chest pain. An ECG is obtained in the emergency department and shows ST segment elevation in leads II, III, and aVF. A sample of blood is taken from the patient, and a new test is used to measure plasma homocysteine levels. The test is repeated 3 times with his blood sample, and the results are 11.8 µmol/L, 9.2 µmol/L, and 13.7 µmol/L (laboratory reference range: 4-14). Which of the following parameters is most likely to be low based on the results of the new test? A.Accuracy B.Precision C.Sensitivity D.Specificity E.Validity

accuracy and validity same same precision and reliabity same same

A 65-year-old woman is treated with gentamicin for an abdominal infection complicated by multidrug-resistant organisms. After a week of treatment, the patient's urine output decreases noticeably, and serum creatinine rises to 2.3 mg/dL. She has no previous kidney disease, and baseline kidney function was normal prior to the initiation of therapy. The patient has remained afebrile for 24 hours. Blood pressure is 130/80 mm Hg and pulse is 80/min. Examination shows moist mucous membranes. There is no rash. Results of urinalysis are as follows: Protein +1 White blood cells 1-2/hpf Red blood cells none Microscopy: granular casts Fractional excretion of sodium is >2%. Histologic examination of the patient's kidneys would most likely show which of the following? A.Focal tubular epithelial necrosis B.Leukocytic infiltration of the glomerular capillaries C.Leukocytic infiltration of the interstitium and tubules D.Preservation of normal renal architecture E.Replacement of glomeruli with collagen

A Aminoglycosides (eg, gentamicin, tobramycin) are bactericidal antibiotics that bind to the 30S ribosomal subunit and inhibit protein synthesis. They are commonly used for severe gram-negative infections but carry a significant risk of acute kidney injury. Aminoglycosides are filtered across the glomerulus and concentrate within the proximal renal tubules, where they impair lysosomal function, protein synthesis, and mitochondrial activity, leading to acute tubular necrosis (ATN). This is visualized histologically as focal tubular epithelial necrosis, often with extensive granular casts that obstruct the tubular lumen and lead to rupture of the basement membrane. Aminoglycoside-induced kidney injury typically manifests within 1 week of therapy initiation. Due to the high intratubular drug concentrations, ATN can occur despite normal serum drug levels. Proximal tubular dysfunction results in loss of resorptive capacity and electrolyte wasting (eg, hypomagnesemia, hypophosphatemia); severe disease can result in Fanconi syndrome (ie, aminoaciduria, glucosuria, uricosuria, phosphaturia). Distal tubular injury may also occur and results in loss of concentrating capacity with polyuria (nonoliguric renal failure). Urinalysis typically demonstrates mild proteinuria with granular or hyaline casts. Consistent with other causes of ATN, the fractional excretion of sodium (FENa) is >2%.

A 52-year-old postmenopausal woman comes to the office for evaluation of several months of episodic abdominal discomfort and nausea, especially after a fatty meal. She has no past medical history and does not use tobacco, alcohol, or illicit drugs. Her BMI is 33 kg/m2. Physical examination shows a soft, nontender abdomen with normal bowel sounds. Liver span is 8 cm. Murphy sign is negative. Abdominal x-ray reveals no calcifications, but abdominal ultrasound shows a small, non-obstructing gallstone. The patient prefers nonoperative management. Which of the following would best treat this patient's condition? A.Bile acid supplement B.Cholestyramine therapy C.Estrogen replacement therapy D.Fenofibrate therapy E.Iron chelation therapy F.Phosphate-binding agent G.Rapid weight loss

A Cholesterol gallstones are the most common type of gallstone. They are primarily composed of cholesterol monohydrate crystals but can contain variable amounts of calcium salts, bilirubin, and mucin. Normally, bile acids and phospholipids solubilize the cholesterol to prevent stone formation. Decreased amounts of bile acids and phospholipids can cause the bile to become supersaturated with cholesterol, allowing it to crystallize and form cholesterol gallstones. Risk factors for stone formation include increasing age, obesity, excessive bile salt loss (eg, terminal ileum disease), and female sex. Cholecystectomy is the preferred treatment for symptomatic gallstones. However, medical therapy is an option in patients refusing surgery or with high surgical risk. Administration of hydrophilic bile acids (eg, ursodeoxycholic acid) reduces cholesterol secretion and increases biliary bile acid concentration. This improves cholesterol solubility and promotes gallstone dissolution. Although the response to medical therapy is good in patients with mild symptoms and small stones, there is a high rate of gallstone recurrence

A 42-year-old man is brought to the emergency department 1 hour after the onset of severe headache, nausea, vomiting, and confusion. The patient has primary (essential) hypertension and chronic kidney disease; he has been prescribed 2 antihypertensive agents but has been noncompliant with therapy. Temperature is 36.8 C (98.2 F), blood pressure is 240/150 mm Hg, heart rate is 90/min, and respirations are 20/min. Ophthalmologic examination shows bilateral papilledema. The lungs are clear to auscultation. Cardiac examination reveals an S4 and no murmurs. Laboratory results are as follows: Today2 months agoHematocrit23%30%Platelets78,000/mm3150,000/mm3Blood urea nitrogen60 mg/dL26 mg/dLSerum creatinine4.5 mg/dL1.8 mg/dLPeripheral smearnumerous schistocytes If renal biopsy is performed in this patient, which of the following pathologic findings is most likely to be found? A.Fibrinoid necrosis of small arteriolar walls B.Focal global glomerulosclerosis C.Granulomatous inflammation of the media D.Homogenous acellular thickening of arteriolar walls E.Interstitial fibrosis and tubular atrophy

A Hypertensive emergency is a condition defined as severely elevated blood pressure (typically >180/120 mm Hg) with evidence of ongoing end-organ damage (eg, renal injury). Hypertensive emergencies typically occur in patients with preexisting essential hypertension and may be induced by medication noncompliance, which can result in a rapid increase in systemic blood pressure. Severe hypertension can cause damage to the vascular endothelium, leading to increased vascular permeability to coagulation factors (eg, fibrinogen), platelet activation, and endothelial cell death. In the kidney, this manifests as malignant nephrosclerosis, which is characterized by 2 distinct histopathologic patterns: Fibrinoid necrosis: Cell death and excessive fibrin deposition within the arteriolar walls is visible as circumferential, amorphous, pink material with smudged, necrotic endothelial cells that lack cytologic detail. Hyperplastic arteriolosclerosis: Over time, activated platelets and injured cells release growth factors, which induce concentric hyperplasia and layering of smooth muscle cells and collagen, resulting in intimal thickening and an "onion-skin" appearance. These changes result in narrowing and/or obliteration of the arteriolar lumen; the resultant erythrocyte fragmentation and platelet consumption can cause a microangiopathic hemolytic anemia that is histologically indistinguishable from hemolytic uremic syndrome. Laboratory abnormalities reflect renal injury (eg, elevated creatinine and blood urea nitrogen) and hemolytic anemia (eg, schistocytes [fragmented red blood cells] on peripheral smear, elevated lactate dehydrogenase and indirect bilirubin). Other end-organ manifestations of hypertensive emergency include papilledema, retinal hemorrhages, pulmonary edema, acute coronary syndrome, encephalopathy, and stroke.

A 32-year-old woman comes to the office for bloating, intermittent diarrhea, and abdominal cramps. Her symptoms have been ongoing for many years and are usually triggered by meals. Medical history and family history are both unremarkable. Vital signs are within normal limits. BMI is 27 kg/m2. The patient is well-appearing, and the abdomen is soft and nontender. Laboratory analysis is unremarkable. The patient is diagnosed with irritable bowel syndrome, and it is recommended that she take loperamide and consume an adequate amount of dietary fiber. After reading about her disorder on the internet, the patient decides to manage her symptoms with a strict vegan diet. Which of the following should be provided as a nutritional supplement for this patient? A.Calcium B.Folic acid C.Magnesium D.Niacin E.Thiamine

A Individuals who adhere to a vegan diet consume only plant-based foods and avoid all meat and other animal-derived products (eg, milk). Adherence to a vegan diet is associated with a decreased risk of type 2 diabetes, hypercholesterolemia, coronary artery disease, and stroke. However, most people obtain a significant portion of daily calcium and vitamin D requirements from dairy products; individuals on a vegan diet often consume less calcium and vitamin D than non-vegans. Supplementation is recommended to avoid long-term consequences, including osteoporosis and bone fractures. Cobalamin (vitamin B12) deficiency can also be seen in vegans, leading to megaloblastic anemia and subacute combined degeneration of the dorsal columns of the spinal cord.

A 32-year-old woman comes to the office with worsening shortness of breath. She has the following arterial blood results: PaO2normal% saturation (SaO2)normalOxygen contentlow Which of the following is the most likely cause of these results? A. Chronic blood loss B. Cyanide intoxication C. High altitude D. Morbid obesity E. Unrepaired Fallot tetralogy

A Oxygen is carried in the blood both as a dissolved gas and in combination with hemoglobin. The total oxygen content of the blood is determined primarily by the amount of hemoglobin in the blood and its percentage of oxygen saturation (SaO2). Dissolved oxygen is not bound to hemoglobin and accounts for a very small proportion of the total oxygen content of blood due to low solubility in plasma. Changes in arterial oxygen tension (PaO2) affect both the SaO2 and the amount of oxygen dissolved in the plasma; however, the change in SaO2 is far more influential on the total oxygen content of the blood. This patient has arterial pO2 (PaO2) and SaO2 within the normal range, but decreased total blood oxygen content (CaO2). If the PaO2 and SaO2 are both normal, the lower total blood oxygen content is most likely secondary to a lower hemoglobin concentration. A common cause of anemia in premenopausal women is chronic blood loss secondary to menstruation.

A 56-year-old man comes to the office due to chronic cough and fatigue. The patient has smoked 2 packs of cigarettes daily since age 18. Physical examination shows cyanosis and expiratory wheezes scattered throughout the lungs. During evaluation for long-term oxygen therapy, his respiratory rate decreases shortly after he begins nasal cannular oxygen supplementation. This patient's reduced respiratory rate is most likely caused by a sudden decrease in stimulation of which of the following sensory receptors? A.Carotid bodies B.Central chemoreceptors C.Juxtaglomerular apparatus D.Pulmonary C fibers E.Pulmonary stretch receptors

A This patient with chronic obstructive pulmonary disease (COPD) developed a decrease in respiratory rate as a result of oxygen supplementation. Oxygen has a minimal effect on respiratory drive unless the arterial partial pressure of oxygen (PaO2) drops below 60-70 mm Hg; therefore, the arterial partial pressure of carbon dioxide (PaCO2) is the major stimulator of respiration in healthy individuals. However, patients with long-standing COPD have decreased sensitivity to PaCO2 (due to chronic CO2 retention) and may also have profound hypoxemia (PaO2 <60 mm Hg); therefore, PaO2 levels can become a significant contributor to respiratory drive. The depth and rate of respirations are controlled by the medullary respiratory center based on input from central and peripheral chemoreceptors and airway mechanoreceptors. Peripheral chemoreceptors found in the carotid and aortic bodies are the primary sites for sensing PaO2 and are stimulated by hypoxemia. When supplemental oxygen is administered, the rapid increase in PaO2 can reduce peripheral chemoreceptor stimulation and decrease the respiratory rate. Reduced respiratory rate is a minor contributor to the oxygen-induced hypercapnia that can occur in patients with COPD; increased ventilation-perfusion mismatch triggered by alleviation of pulmonary vasoconstriction in poorly ventilated areas is the major mechanism.

A 34-year-old woman is found dead in her apartment. Medical history is significant for substance abuse and a prior hospitalization for upper extremity cellulitis. An autopsy examination is performed. A ruptured plaque in the proximal anterior descending artery with an overlying occlusive thrombus is found. Incidentally, a 4-cm hepatic mass in the right lobe of the liver is also found. The mass is lighter than the surrounding liver tissue and appears lobulated with a central, gray-white, depressed stellate scar from which fibrous septae radiate to the periphery. The rest of the liver is normal. No additional abnormalities are noted on autopsy. Which of the following is the most likely diagnosis of this patient's liver lesion? A.Focal nodular hyperplasia B.Hepatic adenoma C.Hepatic hemangioma D.Hepatocellular carcinoma E.Hydatid cyst

A This patient's liver mass is characteristic of focal nodular hyperplasia (FNH), a nonmalignant lesion that most commonly occurs in young women. FNH is thought to result from a hepatic vascular abnormality causing localized hyperperfusion with a secondary hyperplastic response. Classically, the lesions appear as small, solitary, pale nodules composed of cords of normal-appearing hepatocytes and a central stellate scar with fibrous septae that surround abnormally large hepatic arterial branches. FNH is benign and typically does not enlarge, undergo malignant transformation, or rupture; therefore, most cases do not require treatment. These lesions are usually asymptomatic and discovered incidentally during autopsy or abdominal imaging for a different condition. The diagnosis is confirmed most often by observing a characteristic solitary mass on imaging with features indicating the arterial origin of FNH (eg, enhancement with contrast in the arterial phase).

A 32-year-old man is evaluated in the emergency department due to fever, night sweats, and chills over the last several days. The patient has been using intravenous drugs recently as he is "stressed out." He has otherwise been in good health with no medical problems. Temperature is 38.3 C (101 F), blood pressure is 120/80 mm Hg, and pulse is 105/min and regular. Further evaluation reveals aortic valve endocarditis with an intracardiac abscess and small fistula formation between the aortic root and right ventricle. Doppler ultrasound interrogation of the fistula will most likely reveal which of the following blood flow patterns? A.Flow from the aortic root to the right ventricle continuously B.Flow from the aortic root to the right ventricle only in diastole C.Flow from the aortic root to the right ventricle only in systole D.Flow from the right ventricle to the aortic root continuously E.Flow from the right ventricle to the aortic root only in diastole F.Flow from the right ventricle to the aortic root only in systole

A because arotic systole and diastole pressure are always higher than right atrium so the flow will be continous This patient with aortic valve endocarditis has developed an intracardiac fistula between the aortic root and right ventricle. Aortocavitary fistulas are an uncommon complication of bacterial endocarditis caused by extension of the infection from the valve to the adjacent myocardium. Echocardiography with Doppler analysis can be used to detect and quantify shunts in patients with intracardiac fistulas. During the normal cardiac cycle, central aortic pressure (eg, 120/80 mm Hg) is higher than right ventricular pressure (eg, 25/5 mm Hg) during systole and diastole. Consequently, in patients with aortocavitary fistula, Doppler interrogation will most likely demonstrate continuous blood flow from the higher-pressure aortic root to the lower-pressure right ventricle (left-to-right cardiac shunt) (Choices B and C). This can lead to a continuous murmur heard on cardiac auscultation.

A 7-year-old boy is brought to the office by his mother due to facial puffiness that is especially noticeable in the morning. He has a history of mild, intermittent asthma that is well controlled with albuterol as needed. Temperature is 36.1 C (97 F), blood pressure is 98/62 mm Hg, and pulse is 89/min and regular. Physical examination shows bilateral lower extremity pitting edema. Nephrotic-range proteinuria consisting mainly of albumin is revealed on urinalysis. Which of the following mechanisms is the most likely cause of this patient's abnormal laboratory findings? A.Impaired tubular reabsorption of filtered proteins(12%) B.Increased filtration of plasma proteins(82%) C.Inflammation of the urinary tract(1%) D.Necrosis of skeletal muscle fibers(0%) E.Overproduction of low-molecular-weight proteins(2%)

A minimal change disease Albumin is small enough to fit through pores in the GBM and slit diaphragms, but it is not normally filtered through the glomerular filtration barrier due to its negative charge. In patients with minimal change disease, systemic T-cell dysfunction leads to the production of glomerular permeability factor, a cytokine that causes podocyte foot process fusion and decreases the anionic properties of the GBM. Loss of negative charge leads to increased filtration of negatively charged plasma proteins and selective loss of albumin in the urine (selective albuminuria).

A 32-year-old man comes to the office due to progressive fatigue, easy bruising, and recurring episodes of gum bleeding. Physical examination shows several ecchymoses in his lower extremities. Laboratory studies are as follows: Complete blood count Hemoglobin7.8 g/dL Platelets65,000/mm3 Leukocytes3,000/mm3Coagulation studies Prothrombin time22 sec Activated partial thromboplastin time53 sec Plasma fibrinogen134 mg/dL (normal: 200-400 mg/dL) D-dimer4.1 µg/dL (normal: <0.5 µg/dL) Bone marrow biopsy is performed and fluorescence in situ hybridization studies reveal a balanced translocation between the long arms of chromosomes 15 and 17. Which of the following proteins is most likely to be abnormal in the hematopoietic cells of this patient? A. Epidermal growth factor receptor(4%) B. GTP-binding protein(5%) C. Platelet-derived growth factor receptor(14%) D. Retinoblastoma gene product(3%) E. Retinoic acid receptor(71%)

Acute myelogenous leukemia (AML), characterized by failure of immature myeloid precursors (myeloblasts) to differentiate into mature granulocytes, is divided into 8 types (M0 through M7). Acute promyelocytic leukemia (APML), the M3 variant of AML, is associated with the t(15;17) cytogenetic translocation involving the promyelocytic leukemia (PML) gene on chromosome 15 and the retinoic acid receptor alpha (RARA) gene on chromosome 17. Fusion of these 2 genes produces PML/RARA, a chimeric gene (illustrated in this fluorescence in situ hybridization image) that codes for an abnormal retinoic acid receptor, which then inhibits myeloblast differentiation. Abnormal promyelocytes and Auer rods are seen on the smear. The clinical manifestations of AML, including anemia (fatigue, pallor), thrombocytopenia (petechiae, hemorrhages), and neutropenia (fever, opportunistic infections), result from marrow replacement by leukemic cells. As seen in this patient, APML is associated with disseminated intravascular coagulation (DIC), which is characterized by activation of the coagulation cascade. Laboratory findings seen in DIC include thrombocytopenia, elevated D-dimer due to fibrinolysis, and prolonged coagulation profile times (eg, prothrombin time, activated partial thromboplastin time) and low fibrinogen due to consumption. All-trans retinoic acid is used for treatment of APML.

A 45-year-old previously healthy man comes to the office due to worsening muscle pain and weakness over the past 6 months. Carrying heavy items such as groceries has become increasingly difficult, and any prolonged activity causes diffuse muscle cramps. He has also had a 4.5-kg (10-lb) weight gain over this time. The patient takes no medications and does not use tobacco, alcohol, or illicit drugs. Blood pressure is 140/90 mm Hg and pulse is 56/min. BMI is 26 kg/m2. Physical examination shows muscle weakness, predominantly of the shoulder and hip girdles. When the knee reflex is elicited, the quadriceps muscle contracts briskly but relaxation takes longer than normal. There are splitting and peeling of the nails. Serum creatine kinase is elevated. Which of the following tests is most likely to identify the cause of this patient's muscle weakness? A. 24-hour urinary cortisol excretion(12%) B. Acetylcholine receptor antibody titers(10%) C. CT scan of the chest(5%) D. Muscle biopsy(24%) E. Serum TSH level(46%)

An elevated creatine kinase (CK) level suggests a myopathic process with myocyte damage and release of muscle enzymes into the circulation. Common causes of myopathy with elevated CK include hypothyroidism, muscular dystrophies, inflammatory muscle diseases, and medications such as HMG-CoA reductase inhibitors. This patient's fatigue, weight gain, bradycardia, and brittle nails are highly suggestive of hypothyroid myopathy. Decreased thyroid hormone levels impair muscle glycogen and triglyceride use and increase oxidative stress, resulting in myocyte injury. Patients typically presents with myalgias, proximal muscle weakness, decreased exercise tolerance, and cramping. Myoedema (focal muscle contraction at the site of percussion) and delayed relaxation of deep tendon reflexes are characteristic findings that are caused by impaired reuptake of calcium by the sarcoplasmic reticulum. Hypothyroid myopathy and elevations in CK can be the first signs of hypothyroidism and can precede other manifestations by months to years. The diagnosis can be confirmed with an elevated TSH level.

A 41-year-old woman, gravida 0, comes to the office for evaluation of pregnancy. Menarche was at age 12, and her menses recur every 30 days and last 5 days. The patient and her husband have been trying to conceive for the past year and plan intercourse during her fertile window according to the ovulation predictor kit. She has no medical problems, takes no medications, and has no allergies. BMI is 23 kg/m2. Vital signs and physical examination are normal. If fertilization and implantation occurred this cycle, when would the β-hCG level first be detectable in the serum? A. 1 day after fertilization(2%) B. 3 days after fertilization(11%) C. 8 days after fertilization(70%) D. 14 days after fertilization(14%) E. On the day of fertilization(1%)

An ovulation predictor kit measures urinary LH and becomes positive 24 hours before ovulation. Once an oocyte is released from the ovary, sperm may fertilize it for up to 24 hours. After fertilization, the second meiotic division completes and a zygote forms. The zygote travels through the fallopian tube and undergoes mitotic divisions (cleavage), creating smaller cells (blastomeres) that are collectively known as a morula. The morula enters the uterus 3-4 days after fertilization and develops a blastocystic cavity, becoming a blastocyst. The blastocyst implants 6 days after fertilization (Choices A, B, and E), and the outer cell mass (trophoblast) differentiates into the cytotrophoblast and syncytiotrophoblast. The syncytiotrophoblast invades the endometrial connective tissue 6-7 days after fertilization and starts secreting β-hCG, which signals to the corpus luteum in the ovary to continue producing progesterone. Accordingly, β-hCG may appear in the maternal serum 6 days after fertilization at the earliest and may require additional time to rise to a detectable level in serum (typically 8 days). β-hCG is detectable in the maternal serum at <5 IU/L, making serum β-hCG testing the most sensitive method of detecting pregnancy.

A 14-month-old girl is brought to the clinic by her parents due to sores on her feet. Since she took her first steps 3 months ago, she has had blisters involving the soles of the feet, which subsequently break down and heal slowly. The patient had failure to thrive as an infant due to frequent oral ulcerations but otherwise has been healthy and has achieved normal developmental milestones. Examination shows small bullae at the soles with faint erythema and no scarring. Biopsy taken at the margin of a blister shows an intraepidermal cleavage plane. Which of the following most likely contributes to the pathogenesis of this patient's condition? A.Autoantibodies against tissue transglutaminase B.Impaired keratin filament assembly C.Loss of function mutation in filaggrin D.T-cell-mediated hypersensitivity reaction

B Epidermolysis bullosa (EB) is a group of inherited disorders caused by mutations involving intraepidermal and dermoepidermal adhesion complexes in the basement membrane zone. It is characterized by epithelial fragility (eg, bullae, erosions, ulcers) triggered by minor trauma. Light microscopy of skin biopsy typically shows intraepidermal cleavage, although the plane of cleavage can vary based on subtype. The most common form of EB, EB simplex, is caused by mutations in keratin genes that impair the assembly of keratin into filaments. It presents early in life with friction-induced blisters at the palms and soles and other exposed areas. Lesions typically heal with no residual scarring, although patients may have chronic thickening of the skin of the feet. Infants with EB simplex may develop oral blisters with bottle-feeding.

A 43-year-old man comes to the emergency department due to a 3-day history of persistent headaches. The patient has a history of hypertension and has had poor medical follow-up. Blood pressure is 224/115 mm Hg and pulse is 67/min. He appears mildly confused during the physical examination, but no focal neurologic deficits are noted. Funduscopic examination shows bilateral papilledema. Serum creatinine is 1.4 mg/dL. An intravenous medication is initiated that causes arteriolar dilation while also improving renal perfusion and increasing natriuresis. Which of the following agents is most likely being used in this patient? A.Esmolol B.Fenoldopam C.Hydralazine D.Nitroglycerin E.Phenylephrine

B Fenoldopam is a short-acting, selective, peripheral dopamine-1 receptor agonist with little to no effect on alpha- or beta-adrenergic receptors. Dopamine-1 receptor stimulation activates adenylyl cyclase and raises intracellular cyclic AMP, resulting in vasodilation of most arterial beds with a corresponding decrease in systemic blood pressure. Renal vasodilation is particularly prominent and leads to increased renal perfusion, increased urine output, and natriuresis (ie, sodium excretion). This makes fenoldopam especially beneficial in patients with hypertensive emergency and renal insufficiency.

A 19-year-old man comes to the office due to frequent episodes of disorientation, palpitations, tremulousness, and excessive sweating over the past 3 months. He says the symptoms resolve quickly after drinking some juice or a carbonated beverage. The patient was diagnosed with type 1 diabetes mellitus at age 12 and takes short- and long-acting insulin. He has had no dose changes in the past 2 years or diabetic complications and follows up with an ophthalmologist frequently. The patient is a college student and reports some stress due to an upcoming midterm examination. He does not use tobacco, alcohol, or illicit drugs. Vital signs and physical examination are within normal limits. Laboratory results show a hemoglobin A1c of 6.8% and no proteinuria. Which of the following factors most likely precipitated this patient's current symptoms? A.Fructose-rich drink consumption B.Intense exercise C.Mental stress D.Respiratory infection E.Sleep deprivation F.Weight gain

B Glucose uptake by skeletal muscle cells is mediated by glucose transporter type 4 (GLUT4). GLUT4 is translocated to cell membranes and transverse tubules (deep invaginations in the cell membrane) in response to insulin. GLUT4 translocation also occurs during muscle contraction by an insulin-independent mechanism, which is mediated by several cellular factors, including AMP-activated kinase, nitric oxide, and calcium-calmodulin-activated protein kinase. In normal individuals, overt hypoglycemia does not occur with exercise because a drop in blood glucose suppresses insulin release from the beta cells, and counterregulatory hormones (eg, glucagon) increase endogenous glucose production via glycogenolysis and gluconeogenesis. However, patients taking exogenous insulin are vulnerable to exercise-induced hypoglycemia because insulin will continue to be released from the injection site despite falling glucose levels. (Choice A) Patients with hereditary fructose intolerance cannot break down fructose and develop hypoglycemia and vomiting after ingestion of fructose-containing foods. However, this typically presents at the age of weaning (when fructose is introduced) and patients often develop a protective distaste for sweets. Ingestion of fructose-containing juice and soda would worsen the hypoglycemia, not correct it. (Choices C, D, and E) Infection, pain, sleep deprivation, and severe mental/emotional distress tend to cause hyper- rather than hypoglycemia due to increased production of counterregulatory hormones (eg, catecholamines, cortisol). These hormones raise glucose by increasing glycogenolysis and gluconeogenesis and decreasing pancreatic insulin secretion (in patients with residual islet cell function). (Choice F) Weight gain is associated with insulin resistance. Insulin-treated patients who gain weight usually develop hyperglycemia and require increased doses of insulin to maintain control of glucose.

A 15-year-old boy is evaluated for progressive lower extremity weakness. The patient has had difficulty running and keeping up with peers for the past several years and has sprained his ankles on multiple occasions. His father also experienced similar symptoms beginning at the same age and became wheelchair bound. Physical examination shows weakness and atrophy of the distal lower extremity muscles. There is bilateral pes cavus deformity. Ankle reflexes are absent, and sensation to vibration is also absent in the feet. Abnormal function of which of the following is the most likely cause of this patient's condition? A.Muscle dystrophin B.Myelin protein C.Ryanodine receptors D.Synaptic vesicles E.Tau protein

B This adolescent with lower extremity weakness, muscle atrophy, and peripheral neuropathy (ie, decreased vibratory sense, absent ankle reflex) likely has Charcot-Marie-Tooth disease (CMT). CMT is a group of hereditary peripheral neuropathies characterized by mutations in genes coding for peripheral nerve axonal or myelin proteins. The most common subtype, CMT1, occurs due to an autosomal dominant mutation in the PMP22 gene. Manifestations of CMT depend on the underlying genetic abnormality; however, most cases present in adolescence or early adulthood with progressive, symmetric distal muscle atrophy and weakness. Initial manifestations often include frequent ankle sprains or difficulty with exercise. Physical examination typically demonstrates muscle wasting most prominent in the distal lower extremities, with sensory loss, areflexia, pes cavus (ie, high arches), and hammer toes. Upper extremity involvement (ie, intrinsic musculature of the hands) typically occurs later in the disease course.

A 26-year-old man with depressed mental status is brought to the emergency department by police. Due to fear of arrest, he swallowed a handful of pills as the officers approached him. On examination, the patient responds to painful stimuli but is somnolent. Respiratory rate is 6/min and after naloxone bolus infusion increases to 14/min. Lungs are clear to auscultation. Which of the following additional findings were most likely present in this patient on initial assessment? A.Miosis, decreased bowel signs, hypertension B.Miosis, decreased bowel signs, hypotension C.Miosis, increased bowel signs, hypertension D.Mydriasis, decreased bowel signs, hypotension E.Mydriasis, increased bowel signs, hypertension

B This patient developed somnolence and decreased respiratory rate after ingesting unknown pills during a confrontation with the police, which is suggestive of opioid intoxication. This is confirmed by improvement of the respiratory rate on administration of naloxone, an opioid antagonist. Decreased respiratory rate is the best predictor of opioid intoxication and is also a frequent cause of mortality. In addition to depressed mental status and decreased respiratory rate, other classic indications of opioid intoxication include miosis, decreased bowel signs, and decreased tidal volume. Hypotension may also occur due to opioid-induced histamine release from mast cells.

A 43-year-old man comes to the office due to joint pain and stiffness in both hands for the past 6 months. He sometimes awakens with hand pain at night. Over the last year, the patient also has had chronic fatigue and poor sexual performance. He has no history of serious illness and takes no medications. The patient rarely sees a physician. He does not smoke tobacco or drink alcohol. Blood pressure is 126/80 mm Hg and pulse is 80/min. BMI is 25 kg/m2. Hand radiographs reveal bilateral erosions and joint deformities involving the second and third metacarpophalangeal joints. Which of the following is the most likely diagnosis? A.Chronic gouty arthritis B.Hereditary hemochromatosis C.Multiple myeloma D.Reactive arthritis E.Rheumatic fever

B This patient has chronic arthritis, fatigue, and sexual dysfunction, which together suggest hereditary hemochromatosis (HH). HH is an autosomal recessive disease characterized by excessive gastrointestinal absorption of iron, which is then stored as hemosiderin in various tissues. Manifestations include liver disease, skin hyperpigmentation, diabetes mellitus, pituitary hormone deficiencies (eg, central hypogonadism), arthritis, and cardiomyopathy. HH-associated arthritis is thought to be caused by iron deposition in the articular cartilage and synovium, with subsequent free radical damage and crystal deposition. It typically involves the second and third metacarpophalangeal (MCP) joints (in contrast to osteoarthritis of the hands, which most often affects the proximal and distal interphalangeal joints and the first MCP joint). X-ray findings include characteristic deformities (eg, hook-like osteophytes) and deposition of calcium pyrophosphate dihydrate in the articular cartilage (chondrocalcinosis).

A 28-year-old woman comes to the office due to right hand tremors for the past several weeks. The patient has difficulty performing daily activities and feels embarrassed in social gatherings. She has a history of the remitting-relapsing form of multiple sclerosis. There is no family history of tremors. On physical examination, no abnormal hand movement is observed at rest. When the patient is instructed to touch an object on the table, a coarse tremor is observed that gradually increases as the hand moves closer to its target. Dysfunction of which of the following structures is the most likely cause of this patient's tremor? A.Basal ganglia B.Cerebellum C.Motor cortex D.Premotor cortex E.Thalamus

B This patient has developed a large-amplitude (ie, coarse) tremor that worsens as the hand approaches the target, suggesting a cerebellar tremor. Cerebellar tremor can result from multiple pathologies that affect the cerebellum, most commonly multiple sclerosis, as in this patient. Other common causes include stroke, fragile X syndrome, and cerebellar degeneration. Tremors due to cerebellar lesions are classically low frequency (ie, "slow," <5 Hz) and high amplitude (ie, coarse) because they involve the proximal, as well as the distal, muscles. The tremor classically increases in amplitude as an action approaches a target (ie, intention tremor). It is often accompanied by other cerebellar signs, including ataxia, dysmetria, and/or impaired rapidly alternating movements.

A 34-year-old man is evaluated for elevated liver aminotransferases. The patient has no chronic medical conditions but has a history of injection drug use. Family history is notable for liver cirrhosis in his mother. Ultrasonography-guided liver biopsy is performed. Histopathology demonstrates marked panlobular mononuclear cell infiltration that cross into adjacent lobules. Occasional intensely eosinophilic round bodies are seen scattered amongst the hepatic parenchyma. Which of the following is the most likely cause of the latter histopathological finding in this patient? A.Coagulative necrosis due to a toxin B.Cytotoxic T-cell-mediated apoptosis C.Intracellular pigment accumulation D.Intracytoplasmic cytokeratin deposition E.Small foci of fibrinoid necrosis F.Stellate cell activation into myofibroblast

B This patient who has a history of intravenous drug use likely has elevated aminotransferases due to acute viral hepatitis. Most cases are marked by significant panlobular lymphocytic inflammation, which may "bridge" into adjacent hepatic lobules due to collapse of the reticulin framework. Spotty areas of hepatocyte injury are typically seen. Necrotic hepatocytes appear ballooned with pale cytoplasm (ballooning degeneration). Alternatively, hepatocytes may undergo cytotoxic T-cell-mediated apoptosis due to the presence of viral antigens on the hepatocyte surface. These apoptotic cells appear as round, acidophilic (pink on hematoxylin and eosin staining) bodies known as Councilman bodies. Unlike acute viral hepatitis, chronic viral hepatitis is usually characterized by significant inflammation surrounding the portal triad (hepatic artery branch, portal vein branch, bile ductule). Hepatocytes usually have a ground-glass appearance (chronic hepatitis B virus) or an abundance of fat (chronic hepatitis C virus). Intravenous drug use is tightly linked to the transmission of hepatitis B and C virus.

A 42-year-old woman comes to the office with her husband for evaluation of abnormal movements. The patient has been having involuntary facial grimaces and extremity fidgeting. She has no prior medical or psychiatric conditions, but her husband says she has had frequent anger outbursts over the past year. The patient takes no medications. She smoked cigarettes and used illicit drugs for several years during college but not since then. Her father died of a neurological disorder at age 55. Physical examination shows intermittent, fidgety, jerky movements of the hands. MRI of the brain is ordered. Which of the following neuroimaging findings is most likely to be seen in this patient? A.Diffuse ventriculomegaly without sulcal widening B.Enlargement of frontal horns of the lateral ventricles C.Hypoplastic vermis with a dilated fourth ventricle D.Pronounced atrophy of bilateral hippocampi E.Regional atrophy of frontal and temporal lobes

B This patient with chorea (eg, fidgety, jerky, involuntary movements), behavioral abnormalities (eg, outbursts of anger), and a family history of a fatal neurological disease likely has Huntington disease (HD). HD is an autosomal dominant, progressive neurodegenerative disorder that results in the production and accumulation of abnormal huntingtin protein, which is toxic to neurons. Although patients with HD display generalized cortical atrophy as the disease progresses, the most prominent atrophy is seen bilaterally in the caudate nucleus and putamen, leading to the enlargement of the frontal horns of the lateral ventricles. The loss of inhibitory GABAergic neurons in these areas causes decreased regulation of the movement and behavioral centers of the cerebral cortex, resulting in the characteristic constellation of symptoms in patients with HD.

A 68-year-old man comes to the office due to exertional shortness of breath and fatigue, which have progressed over the past year. The patient has a history of hypertension, but medical history is otherwise unremarkable. He is a lifetime nonsmoker. His father died of a "heart attack" at age 70. Blood pressure is 144/74 mm Hg, and pulse is 72/min and regular. Cardiac auscultation reveals a 3/6 ejection-type, late-peaking systolic murmur and a barely audible S2. The murmur diminishes in intensity during the straining phase of the Valsalva maneuver. Which of the following processes underlies this patient's current condition? A.Cardiomyocyte hypertrophy and disarray causing asymmetric septal thickening B.Differentiation of valve fibroblasts into osteoblast-like cells C.Myxomatous valve thickening and elastin fragmentation D.Subendocardial granulomatous lesions with fibrinoid necrosis and subsequent fibrosis E.Thrombus formation on inflamed valvular endothelium

B This patient's soft S2 and late-peaking systolic ejection murmur that decreases in intensity with maneuvers that decrease left ventricular blood volume (eg, abrupt standing, Valsalva straining phase) are consistent with aortic stenosis, which most commonly occurs due to age-related calcific aortic valve disease (CAVD). The early pathogenesis of CAVD is analogous to that of arterial atherosclerosis. The endothelium lining the aortic side of the aortic valve cusps is exposed to the same high pressure and turbulent blood flow as the aortic vascular endothelium. As occurs with atheroma development in the vascular endothelium, these mechanical forces (along with smoking, hyperglycemia, and hyperlipidemia) over time cause damage to the aortic valve cusp endothelium, triggering endothelial dysfunction and the onset of a similar atherosclerotic process. There is subendothelial lipid deposition and infiltration of inflammatory cells (ie, macrophages, T lymphocytes) followed by the release of inflammatory mediators (eg, interleukin-1-beta, transforming growth factor beta-1). Subsequently, there is increased production of proteins involved in tissue calcification (eg, osteopontin). Fibroblasts differentiate into osteoblast-like cells, leading to aberrant bone matrix deposition with progressive valvular calcification and stenosis.

A 58-year-old man comes to the office due to problems urinating. Over the past year, he has had difficulty initiating urination and has noticed a weakened urine stream. He has also had frequent nocturnal voiding. The patient has not passed any visible blood clots and has had no dysuria, fever, recent illnesses, or urinary trauma. He has no other medical conditions and takes no medications. Vital signs are within normal limits. Rectal examination reveals a smooth, enlarged prostate with no tenderness to palpation. Prostate specific antigen is within normal range. Urinalysis reveals 20-30 red blood cells per high power field and no urinary casts. Cystoscopy is performed and shows increased bladder wall trabeculations with normal appearing mucosa. Which of the following is the most likely cause of this patient's hematuria? A.Acquired bleeding disorder B.Friable prostatic blood vessels C.Glomerulonephritis D.Interstitial cystitis E.Transitional cell carcinoma of bladder

B This patient's urinary hesitancy, weakened urinary stream, and nocturia in the setting of an enlarged, nontender prostate raises strong suspicion for benign prostatic hyperplasia (BPH), a common condition in men age >50. In BPH, stromal and glandular hyperplasia in the periurethral and transitional zone can impinge the urethra, leading to progressive urinary voiding (eg, hesitancy, weakened stream) and storage (eg, nocturia, frequency) symptoms. The diagnosis is generally confirmed when examination reveals a smooth, large, non-tender prostate. Microscopic or gross hematuria can sometimes arise in patients with BPH due to the formation of new, friable blood vessels in the area of prostatic hyperplasia. Further examination with cystoscopy is generally warranted to rule out other (potentially life-threatening) causes of hematuria such as bladder cancer and urinary calculi. Cystoscopy in those with BPH often shows increased detrusor wall trabeculations due to detrusor muscle hypertrophy (to generate increased pressure to overcome urethral obstruction).

A 73-year-old woman comes to the physician complaining of progressive, severe pain and discharge from her left ear for the past 2 days. She has had type 2 diabetes for many years and has been noncompliant with her medications and follow-up appointments. On examination, moving or touching the pinna produces extreme pain. Otoscopic examination shows granulation tissue in the left ear canal with a scant amount of discharge. The tympanic membrane is clear, and there is no middle ear effusion. Initial cultures from the ear show a Gram-negative rod. Which of the following microbiological characteristics best describes the infecting organism? A.Comma-shaped and grows well in high pH B.Fast lactose fermenter C.Motile and oxidase positive D.Nonmotile and a lactose nonfermenter E.Requires factors V and X for growth

C

A 6-year-old boy is brought to the ER due to confusion and intractable vomiting. These symptoms began a few hours ago and have been gradually worsening. The patient's mother tells you that the boy contracted a febrile illness from his younger brother two days ago. She states that she gave him "some over-the-counter pills" and it helped his fever. While you talk to the mother, the boy lapses into a coma. The light microscopy of his liver specimen will reveal: A.Apoptosis of hepatocytes B.Periportal inflammatory infiltration C.Microvesicular steatosis D.Centrilobular congestion E.Bile duct proliferation

C 1. Hepatic dysfunction manifests with vomiting and hepatomegaly, but jaundice is rare. Liver function tests reveal increased levels of ALT, AST, ammonia, and bilirubin, and a prolonged PT and PTT. Light microscopy of a liver biopsy shows microvesicular steatosis, the presence of small fat vacuoles in the cytoplasm of hepatocytes (Choice C). No necrosis or inflammation is present in the liver. Electron microscopy findings include swelling, a decreased number of mitochondria and glycogen depletion. 2. Encephalopathy of Reye syndrome is attributed to hepatic dysfunction and the toxic effect of hyperammonemia on the CNS leading to cerebral edema. To avoid the possibility of inducing Reye syndrome, aspirin should not be administered to children under the age of sixteen except for very specific circumstances where it is indicated for the treatment of a serious illness such as in Kawasaki disease where salicylates are a mainstay of treatment.

A 34-year-old female presents with a small bluish lesion under the nail of her right index finger. The lesion is extremely tender to touch. If the lesion is a tumor, its cells of origin are most likely to have which of the following functions: A.Proprioception B.Light touch C.Thermoregulation D.Lymphatic drainage E.Antigen processing F.Nail growth

C A bluish neoplasm occurring underneath the nail bed may be either a glomus tumor (glomangioma) or a subungual melanoma. Both are rare diagnoses. Melanomas are composed of melanocytes, which have the function of pigmentation. Because 'pigmentation' is not among the answer choices, glomus tumor is the most likely diagnosis. A glomangioma is a tumor of the modified smooth muscle cells of a glomus body. Glomus bodies are numerous small, encapsulated neurovascular organs found in the dermis of the nail bed, the pads of the fingers and toes, and the ears. Each glomus body is composed of an afferent arteriole connected to a richly innervated, muscular arteriovenous anastomosis, which is then connected to an efferent vein. Modified smooth muscle cells are arranged in layers around these vascular channels. The role of the glomus body is to shunt blood away from the skin surface in cold temperatures in order to prevent heat loss, and to direct blood flow to the skin surface in hot environments to facilitate the dissipation of heat.

A 63-year-old man comes to the physician after noticing a reddish tinge to his urine for the last couple of days. During evaluation of his hematuria, an abdominal CT scan reveals a left-sided renal mass. Further workup also shows multiple pulmonary and bone nodules. CT-guided biopsy of a peripherally located lung nodule demonstrates renal cell carcinoma. High-dose interleukin-2 (IL-2) is started, and 4 weeks later there is a significant reduction in his tumor burden. Which of the following mechanisms was most likely responsible for regression of his malignancy? A.Anti-angiogenic effect of IL-2 B.Direct cytotoxic effect of IL-2 on the tumor cells C.Enhanced activity of natural killer cells D.IL-2-induced apoptosis of tumor cells E.Increased expression of MHC Class 1 on tumor cells

C Interleukin-2 (IL-2) is produced primarily by helper T cells and is the major growth factor for T lymphocytes. Antigen binding to the T cell receptor stimulates the secretion of IL-2 and the expression of IL-2 receptors (IL-2R). The IL-2/IL-2R (autocrine) interaction then stimulates the growth, differentiation, and survival of antigen-specific CD4+ T cells and CD8+ T cells. IL-2 also promotes the growth of B cells and activates natural killer (NK) cells and monocytes. Increased activity of T cells and NK cells is thought to be responsible for IL-2's anti-tumor effects. IL-2 (aldesleukin) is currently used as immunotherapy for metastatic melanoma and renal cell carcinoma. The IL-2-induced immune response against renal cell carcinoma results in tumor regression in approximately 10% of patients. These responses can persist for many years, and the majority of complete responders remain free of long-term relapses.

A study examined the effect of a low-carbohydrate diet compared to a low-fat diet on body weight. A sample of 150 overweight but otherwise healthy adults from a large city was enrolled in the study and randomly assigned in a 1:1 ratio to either the low-carbohydrate (40 g/d) or the low-fat (<7% saturated fat) diet. At 12 months, a greater body weight change was reported in the low-carbohydrate diet group compared to the low-fat diet group, with a mean difference in body weight change of −3.5 kg (p = 0.01, predetermined significance level = 0.05). Which of the following is the most accurate interpretation of the results of this study? A.The observed mean difference in body weight change of −3.5 kg is not statistically significant B.The probability of observing a mean difference in body weight change of −3.5 kg is 0.01 C.There is a 1% chance of observing a mean difference in body weight change of at least −3.5 kg when no difference between groups is assumed D.There is a 1% chance that an adult on a low-carbohydrate diet will have a body weight change of at least −3.5 kg at 12 months after starting the diet E.There is a 1% chance that the mean difference in body weight change is biased in favor of the low-carbohydrate diet group

C Statistical tests contrast a null hypothesis (H0) and an alternative hypothesis (Ha), in this case: H0 is a claim of no difference between populations (eg, no difference in mean body weight between the low-carbohydrate population and the low-fat population). Ha is a claim of a difference between populations (eg, difference in mean body weight between the 2 populations). Statistical inference uses data from samples (eg, 75 adults in the low-carbohydrate sample, 75 adults in the low-fat sample) to draw conclusions about underlying populations. Sample estimates or differences (eg, sample mean, difference in mean between 2 samples) generally vary with distinct samples (eg, if a different sample of 75 adults was chosen for each group) and may be close but not equal to the underlying population value. One way to account for sampling variation is to calculate the p-value, which is the probability of obtaining a sample value at least as large as the one observed when the population value claimed in H0 is assumed to be true. The magnitude of the p-value compared to a predetermined significance level (eg, 0.05 [or 5%] commonly used as the threshold) determines whether there is convincing evidence against H0. A low p-value (eg, <0.05) occurs when the sample value significantly disagrees with the population value claimed in H0 and provides convincing evidence against H0 (ie, H0 is probably wrong). Results are considered statistically significant. A high p-value (eg, ≥0.05) occurs when the sample value is close to the population value claimed in H0 and provides convincing evidence in favor of H0 (ie, H0 might be correct). Results are considered not statistically significant. In this case, the given p-value = 0.01; therefore, there is a 1% chance (ie, 0.01) of observing a mean difference in body weight change (ie, sample estimate) of at least −3.5 kg between the low-fat and the low-carbohydrate samples when no difference between the populations is assumed (ie, H0 is assumed to be true) (Choice B).

A 55-year-old man comes to the emergency department with recurrent episodes of retrosternal chest pain. The episodes occur during physical activity, usually when he climbs stairs or walks uphill. The patient has no known medical problems and does not use tobacco, alcohol, or illicit drugs. He is given a sublingual tablet and reports rapid relief of the pain. This drug most likely improved this patient's symptoms by causing which of the following hemodynamic changes? A. Decrease in arteriolar resistance(17%) B. Decrease in heart rate(1%) C. Decrease in left ventricular end-diastolic volume(48%) D. Increase in coronary blood flow(29%) E. Increase in coronary perfusion pressure(2%)

C This patient has a classic presentation of chronic stable angina, which is characterized by chest pain that occurs with activity and is relieved with rest or sublingual nitroglycerin. Nitrates exert their effect by direct vascular smooth muscle relaxation that results in: Vasodilation of the peripheral veins and arteries, predominantly venodilation Decreased left ventricular wall stress due to reduced preload (decreased left ventricular end-diastolic volume and pressure) Modest reduction in afterload due to systemic arterial vasodilation Mild coronary artery dilation and reduction of coronary vasospasm This results in decreased myocardial oxygen demand, leading to improved exercise tolerance and relief of angina symptoms.

A 9-month-old girl is brought to the emergency department due to fever and cough. Her symptoms began approximately 2 days ago and have been worsening. The patient is otherwise healthy except for an episode of acute otitis media 3 weeks ago that resolved following a course of oral amoxicillin. She lives with her mother, father, and grandmother who have been in good health. Her mother smokes cigarettes. Temperature is 38.5 C (101.3 F) and respirations are 34/min. Pulse oximetry is 95% on room air. Pulmonary examination reveals mild subcostal retractions and focal crackles in the left lower lobe. Which of the following underlying factors is most likely contributing to this patient's current condition? A.Decreased presence of lung monocytes B.Exaggerated airway reactivity C.Impaired mucociliary function D.Inhibited B cell maturation E.Reduced alveolar surface area

C This patient has fever, cough, and focal crackles, which are findings consistent with pneumonia. Transmission typically occurs via droplet spread of a bacterium (eg, Streptococcus pneumoniae) with subsequent colonization of the nasopharynx and microaspiration into the lungs. Respiratory defense mechanisms often prevent infection of the lower respiratory tract, and the first line of defense is the ciliated epithelium and mucus-producing cells that line the respiratory tract. Pathogens become entrapped in mucus and are transported upward and out of the nasopharynx by the sweeping cilia. Other defense mechanisms include binding by immunoglobulin (eg, secretory IgA, opsonizing IgG and IgM), phagocytosis by alveolar and interstitial macrophages, and cell-mediated cytotoxic activity. Secondhand smoke exposure disrupts natural host defenses and increases the risk of pneumonia, particularly in infants who have a parent that smokes. Cigarette smoke induces mucus overproduction and impairs ciliary function, leading to delayed clearance of pathogens and respiratory secretions. In addition to respiratory infections, children with secondhand smoke exposure are also at increased risk for asthma and wheezing and are more likely to have recurrent otitis media.

A 68-year-old woman comes to the emergency department due to pain in the left upper arm after a fall from standing height. Medical history is significant for hypothyroidism and hypertension. Physical examination shows bruising of the shoulder. Left arm movement is limited due to pain. Plain radiographs reveal a nondisplaced left humerus fracture at the anatomical neck. This patient is at greatest risk for which of the following complications? A.Biceps tendon rupture B.Brachial artery tear C.Humeral head necrosis D.Median nerve injury E.Radial nerve palsy

C This patient suffered a fracture of the proximal humerus at the anatomical neck. Humeral neck fractures commonly occur in falls; risk factors include age >65 and osteoporosis. They often occur as isolated injuries but may also be associated with glenohumeral dislocation. The proximal humerus and glenohumeral joint receive their blood supply via the anterior and posterior circumflex humeral arteries, which are branches of the axillary artery that form an anastomosis encircling the neck of the humerus in the quadrangular space. This anastomosis gives rise to intraosseous arteries that travel back to the humeral head; humeral neck fractures can disrupt this retrograde blood flow, leading to avascular necrosis (osteonecrosis) of the humeral head. Osteonecrosis presents insidiously with shoulder pain, decreased range of motion, and flattening of the humeral head on x-ray.

A 44-year-old man is evaluated in the clinic for occasional chest discomfort that is not consistently related to exertion. The patient's past medical history is significant for hypertension and hyperlipidemia. His grandfather experienced a myocardial infarction at age 50. Coronary CT angiography reveals several nonobstructive atherosclerotic plaques in the coronary arteries. One plaque in the proximal left anterior descending artery appears extensive, has a large hypodense core, and occupies 40% of the lumen. No intervention is performed. One year later, the patient comes to the emergency department with acute severe chest pain and is found to have thrombotic occlusion of the proximal left anterior descending artery. High intraplaque activity of which of the following enzymes most likely resulted in this patient's myocardial infarction? A.Hydroxymethylglutaryl CoA reductase B.Lysyl oxidase C.Metalloproteinases D.Procollagen peptidases E.Prolyl hydroxylase

C This patient with coronary atherosclerosis has likely developed plaque rupture with thrombotic occlusion of the left anterior descending artery, resulting in acute coronary syndrome (unstable angina, myocardial infarction). Patients with coronary atherosclerosis can be asymptomatic but typically develop symptoms of stable angina (exertional chest pain relieved with rest/nitroglycerin) if progressive enlargement of the atherosclerotic plaque causes >70% luminal stenosis. Acute coronary syndrome usually occurs due to plaque rupture, which leads to superimposed thrombosis and vessel occlusion. The likelihood of plaque rupture is typically related to plaque stability rather than plaque size or degree of luminal narrowing. Plaque stability largely depends on the mechanical strength of the fibrous cap: Thin-cap fibroatheromas are generally unstable and more vulnerable to rupture. During the chronic inflammatory progression of an atheroma, the fibrous cap is continually being remodeled. The balance of collagen synthesis and degradation determines the mechanical strength of the cap. Thin-cap fibroatheromas are characterized by a large necrotic core covered by a thin fibrous cap. Activated macrophages infiltrating the atheroma contribute to the breakdown of extracellular matrix proteins (eg, collagen) by secreting metalloproteinases. Ongoing intimal inflammation can destabilize the mechanical integrity of the plaque through release of these metalloproteinases, resulting in plaque rupture and consequent acute coronary syndrome. (Choice A) Hydroxym

44-year-old man comes to the office for follow-up of type 2 diabetes mellitus. He was diagnosed with diabetes 4 years ago and has been taking metformin since then. Six months ago, a new antidiabetic agent was added due to suboptimal glycemic control. Today, the patient is happy because he has lost 7 kg (15.4 lb) without any changes in diet or activity since the new therapy was started. His only symptoms are occasional nausea, vomiting, and a prolonged sensation of fullness after eating. Vital signs are within normal limits with no orthostatic changes. BMI is 34 kg/m2. Physical examination shows normal heart and lung sounds, a nontender abdomen, and normal deep tendon reflexes and sensory examination. Laboratory testing reveals normal renal function studies and hemoglobin A1c of 7.2%. If this patient's weight loss is due to the new medication, which of the following agents is most likely responsible? A.Basal-bolus insulin therapy B.Dipeptidyl peptidase-4 inhibitor C.Glucagon-like peptide-1 agonist D.Sulfonylurea E.Thiazolidinedione

C This patient with uncontrolled diabetes on metformin therapy was started on a second medication to improve his glycemic control. The new agent was associated with significant weight loss, a side effect that is characteristic of glucagon-like peptide-1 (GLP-1) agonists (eg, exenatide, liraglutide). GLP-1 is secreted by intestinal L cells in response to food intake and regulates glucose by slowing gastric emptying, suppressing glucagon secretion, and increasing glucose-dependent insulin release. It acts through cell surface receptors that are coupled with a G protein-adenylyl cyclase system. Native GLP-1 is degraded by dipeptidyl peptidase-4 (DPP-4), but synthetic GLP-1 agonists are resistant to degradation, allowing for a prolonged therapeutic effect. Although insulin secretion is increased by GLP-1 agonists, most patients lose weight due to enhanced satiety and a prolonged feeling of fullness following meals; this effect is also seen in patients who do not have diabetes. GLP-1 agonists are administered parenterally. Adverse effects include nausea and vomiting (due to delayed gastric emptying) and pancreatitis, but the risk of hypoglycemia is low because insulin secretion is not stimulated when circulating glucose levels are normal.

A 14-year-old boy is brought to the clinic by his mother due to changes in vision. For the last few weeks, the patient has been squinting when trying to read the interactive whiteboard in class and has had to sit closer to the front of the room. He has also developed headaches. The patient has no chronic medical conditions and takes no medications. Temperature is 37.2 C (99 F), blood pressure is 116/76 mm Hg, and pulse is 70/min. Examination of the eyes shows equal and reactive pupils. Extraocular muscles are intact, and visual field testing reveals no defects. Visual acuity shows 20/70 in both eyes. His ability to read close text is normal. Which of the following is the most likely cause of this patient's symptoms? A.Chronically increased intraocular pressure B.Disturbance in development of the visual cortex C.Increased anterior-posterior diameter of the eyes D.Loss of accommodating power of the lens E.Refracted image focused posterior to the retina

C this kid has myopia hes nearsighted cant see far. because the anterior posterior diameter of his eye is big so the light doesnt reach retina. This patient with decreased visual acuity and an otherwise normal ophthalmic examination likely has simple myopia (nearsightedness), a refractive error caused by an increased anterior-posterior diameter of the eye. The risk is greatest for those with a positive family history or those of East Asian descent. During the adolescent growth spurt, the axial length of the globe also grows rapidly, making the incidence of new-onset myopia highest in this age group. As the anterior-posterior diameter increases, the focal point of a refracted image lies anterior to the retina. For images to be seen clearly, they must be projected directly onto the retina; therefore, the image displacement in myopia causes blurred vision of long-distance objects and decreased visual acuity on examination. Vision of nearby objects is unaffected and appears clear in myopic patients. The remainder of the examination, including extraocular movements, peripheral field testing, and funduscopic examination, is normal. Management is with prescription glasses with a diverging (concave) lens, which refocuses the refracted image onto the retina. The degree of myopia usually stabilizes by early adulthood.

Experimental gastric function monitoring in healthy subjects before, during and after a meal shows an initial rise, peak, and then decline in gastric acid production. Which of the following helps most to down-regulate gastric secretion after a meal? A.Basal secretion B.Cephalic phase C.Gastrin release D.Intestinal influences E.Receptive relaxation F. Postprandial alkaline tide

Classically, the stimulation of acid secretion within the stomach is separated into three phases: cephalic, gastric, and intestinal. The cephalic phase is mediated primarily by cholinergic and vagal mechanisms, and is triggered by the thought, sight, smell, and taste of food. The gastric phase is mediated by the presence of gastrin (which stimulates histamine secretion and therefore, indirectly, acid secretion), and is triggered by the chemical stimulus of food and distension of the stomach. The intestinal phase is initiated when protein-containing food enters the duodenum, but this phase plays only a minor role in stimulating gastric acid secretion. In fact, intestinal influences are effective in down-regulating gastric acid secretion after a meal. The ileum and colon release peptide YY, which binds to receptors on the endocrine, histamine-containing cells described as enterochromaffin-like (ECLs). Such binding counteracts the cephalic and gastric phases of acid secretion by inhibiting gastrin-stimulated histamine release from ECLs. Other factors that inhibit acid secretion include somatostatin and prostaglandins.

A 65-year-old man comes to the office after his wife insisted he get a checkup. The patient feels well and has no chronic medical conditions but has not seen a physician in many years. He received blood transfusions after a motor vehicle trauma in his 20s. The patient does not use tobacco or illicit drugs but drinks 2 or 3 alcoholic beverages daily. Physical examination reveals no hepatomegaly, ascites, or dilation of the superficial abdominal veins. The remainder of the examination shows no abnormalities. Serology is positive for hepatitis C antibodies. Chronic hepatitis C infection is confirmed with positive HCV RNA testing. After appropriate counseling is provided, combination therapy with sofosbuvir and ledipasvir is planned. This treatment is most likely to help clear the infection through which of the following mechanisms? A.Blocking reverse transcription of viral RNA B.Enhancing antiviral host immune response C.Impairing viral entry into host cells D.Inhibiting viral genome replication and assembly E.Preventing new virion release from infected host cells

D

Scientists studying the kidney's response to hypoperfusion apply a clip to a pig's right renal artery that reduces blood flow to the kidney by about 70%. After 6 months, they perform a right nephrectomy and examine the glomeruli and tubules microscopically. Which of the following cell types would be most likely to undergo hyperplasia as a result of the clip placement? A.Cuboidal epithelial cells of the proximal tubules B.Endothelial cells of the efferent arteriole C.Intraglomerular mesangial cells D.Modified smooth muscle cells of the afferent arteriole E.Squamous epithelial cells of the thick ascending limb of the loop of Henle

D A markedly narrowed renal artery (eg, due to an atherosclerotic plaque or external clip) cannot supply the kidney with enough blood to maintain a normal glomerular filtration rate. The reduced blood flow is sensed by the juxtaglomerular (JG) apparatus, which consists of the macula densa, extraglomerular mesangial cells (ie, Lacis cells), and JG cells. Macula densa cells are tall, narrow cells located in the distal tubule that monitor salt content and tubular flow rate. This information is transmitted to nearby JG cells that are located mainly in the wall of the afferent arteriole. JG cells are modified smooth muscle cells with renin-containing zymogen granules. Significant renal hypoperfusion leads to a compensatory increase in renin secretion by JG cells. This activates the renin-angiotensin-aldosterone system, leading to increased levels of angiotensin II and aldosterone. Long-term renal hypoperfusion, such as caused by renal artery stenosis, leads the JG cells of the affected kidney(s) to undergo hyperplasia as a result of chronic stimulation. When renal artery stenosis is restricted to one side and the contralateral kidney functions normally, chronic kidney disease does not occur as the normal kidney is still able to efficiently filter and excrete waste products (eg, creatinine, urea). However, hypertension can occur in both unilateral and bilateral disease, as secretion of renin by one kidney will lead to systemic vasoconstriction and retention of salt and water by both kidneys.

A 24-year-old woman comes to the office for a routine antenatal ultrasound. She is 19 weeks pregnant by her last menstrual period. This is the patient's third pregnancy, and there have been no complications. Her family history is unremarkable, and both of her children are healthy. The ultrasound reveals a male fetus with bilaterally enlarged fetal kidneys with diffuse small cysts. The amniotic fluid volume is very low. No other anomalies are seen. Which of the following will most likely be present in the newborn after delivery? A.Bladder distension B.Cerebral aneurysm C.Hypertension D.Respiratory distress E.Vertebral anomalies

D Bilaterally enlarged, cystic fetal kidneys and oligohydramnios are findings consistent with autosomal recessive polycystic kidney disease (ARPKD). ARPKD is caused by a mutation in PKHD1, the gene for fibrocystin. Fibrocystin is found in the epithelial cells of both the renal tubule and the bile ducts; deficiency leads to the characteristic polycystic dilation of both structures. Mutations can be inherited in an autosomal recessive pattern or can be spontaneous mutations. Age at presentation is determined by the severity of ARPKD. In its most severe form, ARPKD can be detected on prenatal ultrasound. Oligohydramnios is usually present because amniotic fluid is composed of fetal urine and renal filtration is severely impaired in ARPKD. The reduced amniotic fluid volume leads to characteristic findings (Potter sequence: flattened facies, limb deformities, pulmonary hypoplasia) due to the resultant compression of the fetus. Less severe phenotypes more often present with hepatic complications (eg, hepatomegaly, portal hypertension) and hypertension during childhood or early adulthood. Patients with ARPKD - especially when it presents in infancy - often need dialysis or renal transplant.

A 56-year-old man comes to the cardiology clinic because of fatigue, palpitations, and exertional dyspnea over the last several weeks. On physical examination, his heart rate is irregular and measures 122 beats per minute, while his blood pressure is 110/70 mm Hg. The patient undergoes further work-up, including ECG, radiographic studies, and laboratory studies. His ECG shows atrial fibrillation and trans-esophageal echocardiography reveals a thrombus in a dilated left atrium. In the event of an interruption of blood flow secondary to arterial occlusion, which of the following organs would be least vulnerable to infarction? A.Spleen B.Brain C.Kidney D.Liver E.Heart

D Infarcts in an otherwise normal liver are rare because it has a dual blood supply: the portal vein and hepatic artery. Should the hepatic artery become occluded, the portal venous supply and retrograde arterial flow through accessory vessels (e.g., from the inferior phrenic, adrenal, intercostal arteries, etc.) is typically sufficient to sustain the liver parenchyma. The notable exception is when a transplanted liver undergoes hepatic artery thrombosis. In this case, the liver can develop biliary tree infarction and organ failure because the collateral blood supply is severed during transplantation.

A 37-year-old woman is brought to the emergency department following a motor vehicle collision. She lost control of her car while going around a corner and struck a brick wall head on. The patient has severe pain at the right hip with any movement. Examination shows that the right lower extremity is adducted and internally rotated and appears shorter than the contralateral leg. Range of motion testing at the hip is limited in all directions due to severe pain. X-ray of the hip is shown in the exhibit. Which of the following structures is most vulnerable to involvement from this patient's injury? A.Femoral artery B.Femoral nerve C.Lateral femoral cutaneous nerve D.Sciatic nerve E.Superior gluteal artery

D This patient has a posterior hip dislocation (ie, femoral head displaced posteriorly from the acetabulum), which commonly occurs in head-on motor vehicle collisions in which the knee strikes the dashboard or from a fall in frail, elderly patients. The risk is increased with prosthetic joints. On examination, the leg with a posterior hip dislocation appears shortened and internally rotated, with the hip held in flexion and adduction. Range of motion is limited and painful. Posterior dislocation can be mistaken for intertrochanteric hip fracture, which can also present with leg shortening; however, the leg is typically rotated externally with intertrochanteric fracture due to contraction of the psoas and iliacus without the normal acetabular counterforce. The sciatic nerve runs posterior to the hip joint and is vulnerable to injury with posterior hip dislocation. Findings of sciatic injury include weakness of ankle dorsiflexion, decreased ankle reflex, and decreased sensation in the distribution of the nerve

A 64-year-old man comes to the office due to progressive low back pain for the past several months that is beginning to interfere with his job. The patient describes pain in the lower back and buttocks, which worsens with prolonged standing or walking and improves with lying down. He also has occasional sharp, shooting pain that radiates down his right leg. The patient works as a greeter for a local supermarket and was recently given a written warning for taking too many breaks during his shift. He has a history of hypertension and osteoarthritis. Spine imaging reveals degenerative joint changes and a large osteophyte compressing the spinal nerve root exiting through the right neural foramen between the L5 and S1 vertebrae. Which of the following examination findings is most likely present in this patient due to the nerve lesion? A.Decreased knee reflex B.Decreased rectal tone C.Sensory loss in inguinal region D.Weakness of foot dorsiflexion E.Weakness of hip flexion

D This patient has lumbosacral radiculopathy due to an osteophyte at the L5-S1 neural foramen resulting in compression of the L5 nerve root. Spinal nerves in the lumbosacral spine exit below their corresponding vertebral body level (eg, L5 nerve root exits between L5 and S1 vertebral bodies). However, because of the presence of multiple spinal nerve roots in the lumbosacral area (ie, cauda equina), nerve root compression can occur at different vertebral levels through 2 distinct mechanisms: Spinal spondylosis: Degenerative changes and osteophyte formation can narrow the neural foramina. This leads to nerve root compression as it passes through the neural foramina (in this patient, an osteophyte is compressing the exiting L5 nerve root). Vertebral disc herniation: A tear in the intervertebral disc annulus can lead to herniation of the nucleus pulposus. If this occurs laterally, it can compress the exiting nerve root as it passes through the neural foramina. However, if it occurs more centrally, it can protrude into the spinal canal and compress other nerves in the cauda equina that exit one or more levels below the area of herniation. L5 radiculopathy causes sensory loss and back pain that radiates down the leg in an L5 dermatomal distribution (eg, buttocks, lateral thigh and calf, dorsal foot). Weakness occurs in muscles innervated by fibers from the L5 spinal root, which include the tibialis anterior (foot dorsiflexion and inversion), peroneus (foot eversion), and extensor hallucis longus (great toe extension).

A 65-year-old man comes to the office with a 4-month history of persistent left ear pain that is slowly worsening. He is also having some difficulty swallowing. The patient has no chronic medical conditions but has smoked 1 pack of cigarettes per day for the last 46 years. On examination, the external auditory canal is patent and the tympanic membrane is clear with no middle ear effusion. There is an enlarged lymph node in the left anterior neck. Flexible fiberoptic laryngoscopy reveals an ulcerative mass on the posterior pharyngeal wall of the hypopharynx. Involvement of which of the following nerves is most likely responsible for this patient's ear pain? A.Trigeminal nerve B.Facial nerve C.Vestibulocochlear nerve D.Vagus nerve E.Hypoglossal nerve F.Great auricular nerve

D This patient has otalgia in the setting of a normal ear examination, which is likely referred pain (pain perceived at a different site than its true origin). Further evaluation shows an ulcerative mass in the hypopharynx with an enlarged lymph node, which is consistent with mucosal head and neck squamous cell carcinoma (HNSCC). Many sources of pain in the head and neck can be referred to the ear because several cranial nerves also innervate parts of the ear. Common causes of referred ear pain include temporomandibular joint disease, dental disease, or cancer affecting the tongue base, hypopharynx, or larynx. This patient's referred otalgia is most likely due to his hypopharyngeal carcinoma causing irritation of the vagus nerve (CN X). The vagus nerve has afferent fibers innervating the larynx and hypopharynx and also has a branch that provides afferent sensory input from the external auditory canal. Similarly, the glossopharyngeal nerve (CN IX), which innervates the upper pharynx and base of tongue, also provides sensory innervation to the external auditory canal and can contribute to referred otalgia from tumors in these areas.

A 13-year-old boy is evaluated for abnormal growth. His mother says that he has grown rapidly over the past 10 months and is much taller than his older brother and other children his age. During this time, she has had a hard time finding clothes that fit him and has had to buy new shoes for him each month due to increasing foot size. The patient says that he feels well except for excessive sweating. He has no history of major medical illness and takes no medications. Both parents are of average height and have normally proportioned limbs. The patient is above the 99th percentile for height and at the 90th percentile for weight. On physical examination, he has long extremities with large hands and feet; the lower jaw is protruding. Heart sounds are normal. Sexual development is Tanner stage 4. Which of the following is the most likely underlying cause of this patient's condition? A.Aneuploidy of the X chromosome B.De novo mutation in the fibrillin-1 gene C.Defective growth hormone receptor D.Excessive liver production of IGF-1 E.Excessive thyroxine production F.Increased hypothalamic release of IGF-1 G.Increased pituitary gonadotropin release

D This patient most likely has gigantism, a condition caused by hypersecretion of growth hormone (GH) during childhood. GH is a peptide hormone secreted by the anterior pituitary that acts as both a growth promoter and stress hormone (increasing glucose and free fatty acid levels). Although GH has direct effects on target tissue (eg, chondrogenesis, myocyte protein synthesis), its growth-promoting effects are primarily mediated by insulin-like growth factor-1 (IGF-1), which is released from the liver following stimulation of hepatic GH receptors. IGF-1 binds to a specific receptor tyrosine kinase and stimulates cell growth and proliferation in bone, cartilage, skeletal muscle, and other soft tissues. Gigantism occurs in children and adolescents before fusion of the epiphyseal growth plates, resulting in rapid linear growth along with large hands and feet, thickening of the calvarium, protrusion of the jaw (prognathism), excessive sweating, and oily skin. The adult variant, acromegaly, has no effect on stature because the epiphyseal growth plates have already fused.

A 22-year-old woman comes to the office due to 2 weeks of profound fatigue, which has prevented her from attending college classes. The fatigue was preceded by fevers and sore throat, but these have now improved. The patient is an exchange student from Turkey and has no prior medical conditions. She does not use tobacco, alcohol, or illicit drugs. Vital signs are within normal limits. Oropharyngeal examination shows no abnormalities. There are several enlarged lymph nodes posterior to the sternocleidomastoid muscle bilaterally. Cardiopulmonary examination is unremarkable. The spleen is palpable 1 cm below the left costal margin. No skin rashes or other lymphadenopathy is present. Peripheral blood evaluation shows increased numbers of abnormal white blood cells as shown in the exhibit(atypical lymphocyte) . The observed cytoplasmic granules of these cells are most likely to contain which of the following substances? A.Elastase B.Immunoglobulins C.Major basic protein D.Perforin E.Ribosomes F.Viral capsid

D this girl got MONO This patient's peripheral blood smear reveals a reactive (atypical) lymphocyte. In contrast to normal lymphocytes, which are small, round, and have minimal cytoplasm, reactive lymphocytes are large, scalloped, and have abundant cytoplasm. Reactive lymphocytes are activated, pathogen-specific cytotoxic T cells and natural killer cells that form in response to certain intracellular infections (eg, HIV, cytomegalovirus, toxoplasmosis); they are particularly linked to infectious mononucleosis, a primary Epstein-Barr virus characterized by fever, pharyngitis, adenopathy, splenomegaly, and severe fatigue (as is likely in this patient). Reactive lymphocytes are effector cells that contain cytotoxic granules composed of perforin (creates holes in the infected cell's membrane) and granzymes (enter the cytoplasm of infected cells and trigger cell death), which are released in response to foreign antigens on the surface (MHC class I receptors) of infected host cells.

A 45-year-old previously healthy man is brought to the emergency department after being hit by a car while cycling. The patient is hypotensive and tachycardic on arrival and has comminuted fractures of the pelvis and lower extremities. Volume resuscitation is performed with intravenous fluids and multiple units of packed red blood cells and plasma. Several hours later, the patient develops worsening dyspnea and hypoxia. Chest imaging reveals new bilateral infiltrates. Endotracheal intubation is performed for respiratory support. Despite aggressive resuscitation, the patient dies 24 hours later. Autopsy examination of the lungs shows protein-rich fluid within the alveolar airspaces. Breakdown of the barrier formed by which of the following cells most likely contributed to this patient's pulmonary pathology? A.Alveolar macrophages B.Ciliated epithelial cells C.Club cells (previously Clara cells) D.Goblet cells E.Type I pneumocytes F.Type II pneumocytes

E ARDS

A 9-year-old girl is brought to the office by her mother for evaluation of frequent stomach aches. The mother says that her daughter has always had a "sensitive stomach" but that it has gotten worse since she started going to a new school a few months ago. The girl has been missing class due to being in the nurse's office multiple times per week. The patient's symptoms improve when her mother comes to get her. In response to the doctor's questions about the girl's peer relationships, the mother replies, "She has friends but never enjoys sleepovers." When the patient is at a friend's house, she often complains of stomach pain and nausea. She occasionally has nightmares about being left alone and wakes up crying. Although both parents are healthy, the patient says she worries about them "dying of a terrible illness, like cancer." Abdominal examination shows no abnormalities. Which of the following is the most likely diagnosis in this patient? A.Adjustment disorder with anxious mood B.Dependent personality disorder C.Generalized anxiety disorder D.Illness anxiety disorder E.Separation anxiety disorder F.Social anxiety disorder G.Somatic symptom disorder

E Although separation anxiety is common in young children and in older children at times of change or stress, this patient's distress is excessive and impairing when she is separated from her mother. Her inability to sleep away from home, abdominal pain when separated from her mother, worries about losing her parents, and nightmares about being left alone are consistent with separation anxiety disorder. Children with this disorder commonly experience physical symptoms (eg, headaches, stomach aches, nausea) when separation occurs or is anticipated, and have repeated nightmares involving the theme of separation. According to DSM-5, symptoms must be present for ≥4 weeks in children and ≥6 months in adults.

In normal female development, non-fusion of the urethral folds forms the labia minora and the vestibule of the vagina. In males, non-fusion of the urethral folds would most likely result in which of the following? A.Bifid scrotum B.Cryptorchidism C.Epispadias D.Hydrocele of the testis E.Hypospadias

E Differentiation and development of the external genitalia occurs during weeks 8-15 of gestation. In females, the urethral (urogenital) folds do not fuse and ultimately form the labia minora and the vestibule of the vagina. In males, the urethral folds fuse to form the ventral aspect of the penis and the penile raphe, which serve as the anterior wall of the urethra. Incomplete fusion of these folds in a male would result in an abnormal opening of the urethra at a location proximal to the distal tip of the glans penis. Depending on the degree of nonunion, the urethral opening can be anywhere from the perineum to just proximal to the glans penis. This condition is known as hypospadias and can generally be repaired surgically to allow normal urination and sexual activity.

A 62-year-old man is brought to the emergency department by a friend due to severe shortness of breath. He is agitated and gasping for air and is unable to provide history. His friend says the patient has a history of alcohol abuse and attends Alcoholics Anonymous meetings. He is unaware of any other medical history. Temperature is 36.7 C (98 F), blood pressure is 110/70 mm Hg, pulse is 104/min, and respirations are 32/min. Due to worsening respiratory distress, immediate endotracheal intubation is performed and mechanical ventilation is begun. The patient is admitted to the intensive care unit and, despite appropriate therapy, dies several hours later. Autopsy examination is performed, and histopathology of the lungs reveals engorged pulmonary capillaries and intraalveolar, acellular, pink material that is more prominent at the bases. This patient most likely had which of the following conditions? A.Aspiration pneumonia B.Centriacinar emphysema C.Hypersensitivity pneumonitis D.Pulmonary arterial hypertension E.Pulmonary edema F.Pulmonary infarction

E Engorged alveolar capillaries reflect increased pulmonary venous pressure, and pink, acellular material within the alveoli results from transudation of fluid plasma across the alveolar-capillary membrane. These histopathologic findings are most consistent with acute pulmonary edema caused by increased alveolar capillary hydrostatic pressure from left-sided heart failure. This patient likely had dilated cardiomyopathy due to chronic alcohol abuse and had an acute exacerbation of heart failure, resulting in cardiogenic shock with pulmonary edema and acute respiratory failure.

A 26-year-old Caucasian female is found to have a single amino acid substitution (glutamine for arginine) near the protein C cleavage site in her coagulation factor V gene product. The patient is at greatest risk for developing which of the following conditions? A.Petechiae and ecchymoses B.Recurrent hemarthroses C.Renal artery stenosis D.Splenic infarction E.Pulmonary thromboembolism

E Factor V Leiden causes thrombophilia through two pathophysiological mechanisms. In normal hemostasis, activated protein C (APC) restricts clot formation by proteolytically inactivating factors Va and VIIIa. Factor Va Leiden has reduced susceptibility to cleavage by APC, however. Because factor Va is a cofactor in the conversion of prothrombin to thrombin, persistently circulating factor Va Leiden results in increased thrombin production. Additionally, factor V Leiden is unable to support APC anticoagulant activity. This combination of increased coagulation and decreased anticoagulation produces the hypercoagulable state seen in those with factor V Leiden.

A 28-year-old woman, gravida 2 para 2, comes to the office with worsening shortness of breath over the past week. She had a recent episode of hemoptysis. The patient has also had ongoing vaginal bleeding after an uncomplicated vaginal delivery of her son 9 weeks ago. She has no bleeding elsewhere, and she has not resumed sexual intercourse. On physical examination, the patient's uterus is enlarged and the adnexa are normal. Laboratory studies show markedly increased β-hCG levels. Chest radiograph shows multiple bilateral lung nodules. Which of the following would most likely be found on endometrial curettage in this patient? A.Bundles of smooth muscle cells with fibrosis B.Diffusely hydropic chorionic villi C.Fetal tissue with triploid karyotype D.Glands lined by atypical columnar epithelial cells E.Proliferation of cytotrophoblasts and syncytiotrophoblasts

E Gestational choriocarcinoma is a malignant tumor that arises from the trophoblast. It is most commonly preceded by a normal pregnancy but can occur following any pregnancy (molar, ectopic, aborted). The tumor causes abnormal vaginal bleeding, uterine enlargement, and significantly increased β-hCG levels. Choriocarcinoma is aggressive and rapidly invades the uterine wall with hematogenous spread thereafter. The lungs are the most common site of distal metastasis, and presenting symptoms can include shortness of breath and hemoptysis. On macroscopic examination, gestational choriocarcinoma is seen as a bulky intrauterine mass that is usually soft and yellow-white, with extensive areas of necrosis and hemorrhage. Histologically, it is composed of abnormal proliferation of mononuclear cytotrophoblasts (red arrows) and multinuclear syncytiotrophoblasts (green arrows). No villi are present.

A 74-year-old woman comes to the emergency department due to 2 days of left leg pain and numbness. She has a history of atrial fibrillation and hypertension. Her left lower limb is pale and cold to palpation below the knee. Arterial Doppler ultrasound shows an occluding thrombus, and a left femoral embolectomy is performed. Post-procedure dorsalis pedis and posterior tibial pulses become palpable. Several hours later, the patient begins to experience severe left calf pain. On repeat examination, the left calf is swollen, tense, and tender, and the pain worsens with dorsiflexion of the left foot. The distal pulses are also absent. The physician suspects injury of the reperfused muscles in part due to overproduction of reactive oxygen species. Which of the following enzymes is most likely to help neutralize these toxic molecules? A.Cytochrome oxidase B.Cytochrome P450 C.Myeloperoxidase D.NADPH oxidase E.Superoxide dismutase

E The vignette describes acute compartment syndrome secondary to reperfusion injury. Restoration of arterial blood flow to an affected limb floods ischemic tissue with oxygen. Molecular oxygen reacts with xanthine oxidase, NADPH oxidase, and nitric oxide synthase to form reactive oxygen species (ROS) such as superoxide, hydroxyl radicals, and singlet oxygen. ROS disrupt cellular functions leading to DNA mutations, protein synthesis disruption, and membrane lipid peroxidation. Antioxidant enzymes, such as superoxide dismutase, glutathione peroxidase, and catalase, convert ROS to oxygen and water, neutralizing their capacity for cellular damage. ROS generated during cellular respiration are typically neutralized by antioxidants before they become problematic. However, in the postischemic state, the production of ROS exceeds the neutralizing capabilities of antioxidant enzymes (high oxidative stress), leading to increased cell injury and death.

A 24-year-old man comes to the office due to severe pain in the posterior aspect of the right foot that interferes with walking. For the last several months, the patient has also experienced a dull backache and morning stiffness that is relieved by over-the-counter ibuprofen. Family history is insignificant, and he does not use tobacco, alcohol, or illicit drugs. Which of the following is the most probable location of the pathologic process responsible for this patient's foot pain? A.Articular cartilage B.Cortical bone C.Periarticular bursae D.Synovial membrane E.Tendon insertion site

E This patient has inflammatory back pain (ie, onset at age <40, chronic/insidious pain and stiffness that are better with activity and worse with rest), which is likely due to ankylosing spondylitis (AS). In AS, the inflammatory process is thought to originate in the gut due to an altered intestinal biome and defects in the mucosal barrier; the resulting inflammatory response leads to upregulation of cytokines (primarily tumor necrosis factor-alpha and IL-17), with activation of innate immunity and migration of immune cells to the skeleton. The resulting skeletal manifestations are most prominent in areas of mechanical stress, primarily at the insertions of tendons, ligaments, and joint capsules (collectively, entheses) on bone. Enthesitis is a relatively specific finding in AS and other forms of spondyloarthritis and is characterized by local pain, tenderness, and possible swelling. Common clinical syndromes include Achilles tendinitis, plantar fasciitis, and dactylitis ("sausage digits"). Achilles tendinitis affects the site of insertion of the Achilles tendon on the calcaneus and typically presents with severe pain at the posterior aspect of the heel.

A 4-year-old boy is being evaluated for failure to thrive, shortness of breath, and exercise intolerance. The parents first became aware of his symptoms after the patient started preschool a few weeks ago. His teacher mentioned that the patient seemed to tire faster than the other children when playing outside. Cardiac examination shows bounding peripheral pulses and a palpable thrill below the clavicle near the left upper sternal border. On auscultation, a continuous murmur is best heard over the same region. After discussion with the parents, a thoracotomy is scheduled to correct the patient's condition. During the procedure, the surgeon should plan on ligating a derivative of which of the following embryologic structures? A.Bulbus cordis B.Fourth aortic arch C.Primitive atria D.Sinus venosus E.Sixth aortic arch

E This patient most likely has a patent ductus arteriosus (PDA). The ductus arteriosus is an embryonic derivative of the sixth aortic arch that allows fetal blood to pass directly from the pulmonary artery to the proximal descending aorta (bypassing the pulmonary circulation). This vessel usually closes shortly after birth, secondary to decreased prostaglandin E2 (PGE2) levels and increased oxygen concentration. Patency of the ductus after birth results in a left-to-right shunt that can cause left ventricular volume overload and symptoms of heart failure (eg, failure to thrive, respiratory distress). Physical examination will show a continuous "machinery-like" murmur and palpable thrill over the left infraclavicular region due to turbulent blood flow through the PDA. Pharmacologic closure of a PDA can be achieved by PGE2 synthesis inhibitors (eg, indomethacin) in premature infants. However, older patients usually require surgical ligation or percutaneous PDA occlusion.

An 18-year-old girl with a history of type 1 diabetes mellitus comes to the emergency department after several hours of nausea, vomiting, and abdominal pain. The patient reports that she is on vacation visiting friends and has not taken insulin in the last 2 days. Physical examination shows breath with fruity odor and dry mucosal membranes. Laboratory studies show a blood glucose level of 452 mg/dL and high anion gap metabolic acidosis. Urinalysis is positive for ketones. It is determined that the patient's insulin deficiency is causing increased production of gluconeogenic precursors that are subsequently converted to glucose in the liver. Which of the following enzymes is most likely to provide these precursor substrates? A. Acetyl CoA carboxylase(41%) B. Acyl CoA synthetase(25%) C. ATP citrate lyase(4%) D. Glucose-6-phosphate dehydrogenase(8%) E. Glycerol kinase(19%)

E glycerol kinase takes glycerol and converts it into glycerol 3 phosphate which then gets converted into DHAP and can can be used as precursor for gluconeogenesis and glycolysis This patient in diabetic ketoacidosis is experiencing increased triglyceride breakdown in adipose tissue due to her insulin deficiency. Triglycerides stored in adipose tissue are metabolized to free fatty acids and glycerol by hormone-sensitive lipase in response to low insulin and high catecholamine levels. Adipocytes are unable to metabolize glycerol, so it is secreted into the circulation and transported to the liver, where it is phosphorylated to glycerol-3-phosphate by glycerol kinase. Glycerol-3-phosphate is subsequently converted by glycerol-3-phosphate dehydrogenase to dihydroxyacetone phosphate (DHAP), which can be used to produce glucose through gluconeogenesis.

A 27-year-old man comes to the emergency department with progressive right knee swelling and pain. He has no history of trauma to the area. The patient has no other medical conditions and takes no medications. X-rays reveal a large lytic lesion involving the proximal tibia with extensive soft-tissue swelling. After additional confirmatory testing, the patient undergoes a right-sided, above-knee amputation. Histologic examination of the resected mass is shown in the exhibits. Which of the following is the most likely diagnosis? A.Chondrosarcoma B.Ewing sarcoma C.Metastatic adenocarcinoma D.Osteoid osteoma E.Osteosarcoma

E look at the histology but know that its not ewing because ewing is small round neuroectodermal cells resemble lymphocyte This patient's spindle-shaped tumor cells admixed with bone and osteoid indicates osteosarcoma, the most common primary bone tumor in children and young adults. Most cases arise in the metaphyses of long bones (eg, proximal tibia), the location of the growth plate and site of greatest bone proliferation. Patients typically present with pain and soft-tissue swelling, and x-ray usually reveals a lytic bone lesion. Osteosarcomas arise from a malignant mesenchymal stem cell that generates cartilage, bone, or fibrous tissue. Therefore, the diagnosis is confirmed when histopathology reveals neoplastic spindle-shaped stromal cells admixed with tumor osteoid and thin trabeculae of bone.

A 25-year-old man is brought to the emergency department after being found unresponsive. He has a history of schizoaffective disorder and previous suicide attempts by prescription medication overdose. The patient was recently started on haloperidol and sertraline. Temperature is 40.6 C (105 F), blood pressure is 180/98 mm Hg, and pulse is 112/min. On examination, the patient is obtunded and diffusely rigid without clonus. Laboratory studies show serum creatinine of 3.2 mg/dL, creatine kinase of 75,000 U/L, and leukocytes of 16,000/mm3. The antidote for this patient's current condition most likely has which of the following mechanisms of action? A.Activation of presynaptic voltage-gated calcium channels of motor neurons B.Antagonism of peripheral and central serotonin receptors C.Blockade of central dopamine D2 receptors D.Inhibition of acetylcholinesterase at the neuromuscular junction E.Inhibition of calcium ion release from sarcoplasmic reticulum of skeletal muscle

E neuroleptic malignant syndrome give dentrolene. if it was seratonin syndrome youd have hyperreflexia and myoclonus This patient has neuroleptic malignant syndrome (NMS) likely caused by an overdose of haloperidol. NMS is a life-threatening adverse reaction most commonly resulting from the use of antipsychotics (neuroleptic) medications, which block dopamine receptors in the brain. NMS presents clinically with diffuse muscle rigidity, high fever, autonomic instability (hypertension, tachycardia), and altered sensorium. Laboratory findings include creatine kinase elevation due to rhabdomyolysis, which can result in acute renal failure. The mainstay of treatment is to discontinue the causative agent and provide supportive care (eg, intravenous hydration). In severe cases, dantrolene can be administered, which antagonizes ryanodine receptors and inhibits calcium release from the sarcoplasmic reticulum. Bromocriptine, a dopamine agonist, has also shown clinical benefit in NMS.

A 70-year-old man comes to the clinic due to intermittent leg pain and difficulty walking. He describes the pain as mild-to-moderate, deep, and lasting throughout the day. The patient has a history of osteoarthritis in his hands for which he takes over-the-counter nonsteroidal anti-inflammatory drugs, but says that he never has had problems with his knees. Physical examination shows point tenderness over the right tibia. Laboratory studies show elevated serum alkaline phosphatase. Bone biopsy shows haphazardly oriented segments of lamellar bone with prominent cement lines. The initial phase of this patient's disorder involves increased activity of which of the following cell types? A.Chondrocytes B.Endothelial cells C.Fibroblasts D.Osteoblasts E.Osteoclasts

E pagets disease of the bone 3 phases 1) osteoclastic 2) mixed 3) osteoblastic

An autopsy is performed on an 8-month-old infant after his unexpected death. Prior to death, the patient had a 1-month history of poor feeding and intermittent fever, which progressed to severe lethargy and coma. The infant was adopted soon after birth, and there is limited information about his family history beyond documentation of an uncomplicated pregnancy and delivery at term gestation. Brain biopsy reveals leptomeningeal inflammation and is positive for Enterovirus by PCR. This patient would have been most likely to have which of the following laboratory findings? A. Abnormal dihydrorhodamine test(12%) B. Absent peripheral neutrophils(9%) C. Absent tissue eosinophils(1%) D. Decreased total hemolytic complement(7%) E. Low circulating B lymphocyte count(48%) F. Reduced serum IgG2 immunoglobulin levels(21%)

Enterovirus is a common infection in children that typically causes self-limited disease, such as herpangina, hand-foot-and-mouth disease, or in some cases, aseptic meningitis. However, because neutralizing antibodies are required to clear Enterovirus, severe life-threatening infection can occur in infants with a primary humoral immunodeficiency such as X-linked agammaglobulinemia (XLA). XLA, or Bruton agammaglobulinemia, is due to a mutation in the BTK gene encoding Bruton tyrosine kinase. A defect in this protein prevents pre-B cells from differentiating and exiting the bone marrow. Therefore, patients have low circulating B lymphocytes and low immunoglobulin levels. In addition to enteroviral infection, XLA also increases the risk for recurrent sinopulmonary infections by encapsulated bacteria (eg, Streptococcus pneumoniae, Haemophilus influenzae type b), which require antibody opsonization for phagocytosis. Patients with impaired humoral immunity are also predisposed to gastrointestinal infections (eg, Salmonella, Campylobacter), as well as chronic giardiasis (due to low secretory IgA).

A 35-year-old, previously healthy man is evaluated immediately after accidental exposure to ionizing radiation at a fluoroscopy manufacturing facility. The patient is currently asymptomatic. Vital signs are normal. Physical examination shows no abnormalities. If radiation exposure is significant, which of the following cells are most likely to be affected first? A.Bladder epithelial cells B.Cardiac myocytes C.Intestinal crypt cells D.Renal proximal tubular cells E.Skeletal muscle cells F.Type II pneumocytes

Exposure to ionizing radiation causes cell injury directly through DNA double-strand breakage and indirectly through the generation of free radicals that can damage DNA and other cellular components. Significant DNA damage results in the upregulation of p53, which inhibits replication and induces apoptosis of the affected cell. The effects of ionizing radiation vary based on the rate of cellular division. Highly proliferative cells (eg, stem cells) have less time to repair radiation-induced DNA damage before cell death occurs. Therefore, these cells are highly sensitive to ionizing radiation and most likely to be affected first following exposure. Intestinal crypt stem cells divide continuously to produce new intestinal epithelial cells, which have a short life span and are completely renewed every 3-5 days. Rapid denudation of the gastrointestinal mucosa can occur if the radiation dose is significant, resulting in nausea/vomiting, diarrhea, and gastrointestinal bleeding. Other cells that are highly sensitive to the effects of ionizing radiation include the following: Bone marrow: hematopoietic progenitors (pancytopenia) Gonads: spermatogonia (infertility) Skin: basal keratinocytes, hair follicle stem cells (desquamation, hair loss)

A group of researchers is studying population-based screening interventions to reduce mortality from abdominal aortic aneurysm. This disorder develops primarily in elderly patients and is often asymptomatic until an acute rupture event, which is frequently fatal. Screening of high-risk patients for abdominal aortic aneurysm is found to reduce mortality. Which of the following risk factor combinations would likely define the highest-risk group for screening purposes? A.Diabetes, smoking B.Female, hyperlipidemia C.Female, hypertension D.Hyperlipidemia, hypertension E.Male, diabetes F.Male, smoking

F Major risk factors for development of AAA include: Age >65 Smoking (up to 15-fold risk increase) Male sex Smoking appears to be associated with increased inflammatory infiltrates and formation of reactive oxygen species (eg, superoxide anion) in the aortic wall. In light of this, screening for AAA with ultrasonography is recommended for men age 65-75 who have ever smoked. The risk of rupture increases with aneurysm diameter, and surgical or endovascular repair is considered for aneurysms ≥5.5

A 46-year-old man is diagnosed with resistant hypertension. He had a comprehensive workup for secondary hypertension, which was unrevealing. The patient has tried multiple antihypertensive medications without significant effect. Past medical history is otherwise unremarkable. He agrees to enroll in a trial of a new long-acting medication that causes selective direct relaxation of the smooth muscle of arterioles but does not affect the veins. Which of the following adverse effects is most likely to be caused by the drug during the clinical trial? A.Angioedema B.Bradycardia C.Cold extremities D.Decreased cardiac output E.Persistent cough F.Sodium and fluid retention G.Transient hypertension

F Selective arteriolar vasodilators (eg, hydralazine, minoxidil) lower blood pressure by reducing systemic vascular resistance. However, this effect is limited by subsequent stimulation of baroreceptors with resulting reflex sympathetic activation. This leads to increased heart rate, contractility, and cardiac output (Choices B and D). In addition, sympathetic stimulation of the renin-angiotensin-aldosterone axis results in sodium and fluid retention with peripheral edema. These effects offset much of the blood pressure lowering effect of these drugs and limit their long-term efficacy. These agents are rarely used as monotherapy for chronic management of hypertension. However, they are useful acutely for patients with severely elevated blood pressure. They can also be given in combination with sympatholytics and diuretics to mitigate the side effects and provide synergistic blood pressure lowering in patients with resistant hypertension.

It is suggested that some unknown factors may specifically interrupt the normal migration of neural crest cells to the intestine. This migration process is typically complete by the 12th week. If progress is interrupted during the final week of migration, which of the following organs are most likely to be affected? A.Duodenum B.Jejunum C.Ileum D.Cecum E.Sigmoid colon F.Rectum

F rectum and anus always involved. sigmoid colon only 75% of the time Neural crest cells start migrating to the intestinal wall very early during embryonic development. They give rise to ganglion cells of the submucosal (Meissner) and myenteric (Auerbach) plexi of the bowel wall. These parasympathetic ganglia are responsible for intestinal peristalsis. Neural crest cells move caudally along the vagal nerve fibers. They are present in the wall of proximal colon at 8th week of gestation and in the rectum by 12th week. The arrest of migration of neural crest cells causes Hirschsprung disease, in which a segment of colon is deprived of ganglion cells. Since neural crest cells migrate caudally, the rectum is always involved in Hirschsprung disease. The absence of ganglion cells in the colonic wall causes the affected segment to be narrowed because it cannot relax. The passage of intestinal contents through this area is difficult, and compensatory dilatation of proximal areas of the colon occurs. Newborns with Hirschsprung disease fail to pass meconium within 48 hours of birth. They also demonstrate other symptoms of intestinal obstruction, such as bilious vomiting and abdominal distention. The bowel is filled with stool, but the rectum is empty; the tone of the anal sphincter is usually increased. Rectal biopsy reveals an aganglionic segment of bowel.

Studies conducted following an influenza epidemic that affected the inhabitants of several countries determine that the cause is an antigenically novel virus strain carrying the animal-strain hemagglutinin and neuraminidase surface molecules. Further analysis reveals that reassortment of the genetic materials occurred during coinfection with human influenza virus and swine influenza virus, leading to the development of some progeny that contain genetic materials from both viruses. Which of the following viruses can undergo a similar process? A.Hepatitis C virus B.Human papillomavirus C.Measles virus D.Poliovirus E.Rotavirus

Genetic shift in the influenza A virus involves reassortment of genomic segments of the human strain with genomic segments of the animal strain, typically when both strains coinfect birds (avian) or pigs (swine). The resulting new strain can cause a new influenza epidemic or pandemic if it has dramatically altered viral surface glycoproteins (eg, hemagglutinin). The key structural feature that enables influenza viruses to undergo such relatively rapid genetic shifts through reassortment is the segmented nature of their genomes. Members of another virus family with a segmented genome could also undergo such reassortment - as opposed to the less radical, slower genetic drift that occurs via point mutations in all viral families. Among the viruses listed, only rotavirus has a segmented genome. Segmented viruses include orthomyxoviruses, reoviruses, bunyaviruses, and arenaviruses.

A 5-year-old boy with severe, recurrent respiratory infections is undergoing evaluation. Sputum studies reveal intracellular bacteria. Further testing shows that the patient's T cells lack the IL-12 receptor. Supplementation with which of the following substances would most likely improve this patient's condition? A. Early complement components(2%) B. GM-CSF(5%) C. Immunoglobulins(7%) D. Interferon-gamma(78%) E. Interleukin-4(5%)

Helper T cells that have not yet contacted antigens are called naive (Th0) cells and have T-cell receptors (TCRs) and CD4 proteins on their surface. An antigen-MHC class II complex on an antigen-presenting cell (eg, macrophage, dendritic cell) can activate a naive Th cell by interacting with both TCR and CD4. The activated T cell then differentiates into either a Th1 or Th2 cell. If the antigen is presented by a macrophage, the macrophage will produce IL-12, which stimulates differentiation into Th1 cells. A deficiency of IL-12 receptors on naive T cells prevents differentiation into Th1 cells, which produce interferon-gamma (IFN-γ). IFN-γ is responsible for activation of the macrophage and CD8+ cytotoxic response against intracellular organisms, such as mycobacteria. Therefore, individuals with an IL-12 receptor deficiency are susceptible to severe, persistent mycobacterial infections. Administration of IFN-γ improves the immune response to mycobacterium in these patients.

Atherosclerotic lesions of the coronary arteries can limit blood flow to the myocardial regions supplied by the affected vessels. Certain medications can cause a redistribution of blood flow away from the ischemic areas, exacerbating existing myocardial ischemia. Which of the following drug effects is most likely to produce this phenomenon? A. Coronary arteriolar dilation(59%) B. Decreased myocardial contractility(5%) C. Epicardial vessel dilation(11%) D. Negative chronotropic effect(4%) E. Systemic venous dilation(18%)

Hemodynamically significant atherosclerotic lesions (>50% occlusion) reduce the amount of blood supplied by a coronary vessel to the corresponding myocardial territory. In response, the myocardium releases locally acting, endogenous substances (eg, adenosine, nitric oxide) that cause dilation of coronary arterioles, leading to a downstream reduction in pressure and recruitment of additional blood flow to that region. Arterioles that supply areas of significantly ischemic myocardium can become maximally dilated at rest, limiting their capacity to further increase blood flow. Pharmacologic stress agents (eg, adenosine, dipyridamole) are used during myocardial perfusion imaging to simulate the generalized coronary arteriole dilation caused by exercise to assist in identifying areas of ischemic myocardium. Administration of these drugs will result in no change in blood flow to ischemic myocardium and a significant increase in blood flow to nonischemic myocardium; the amplified relative difference in blood flow allows for easy detection of ischemic myocardial areas. When recruitment of additional upstream blood flow is limited (eg, due to a proximal atherosclerotic lesion), these drugs can lead to an absolute decrease in blood flow to ischemic myocardium, as most of the limited blood supply is redirected toward the newly vasodilated areas of nonischemic myocardium. This redistribution of blood flow is referred to as coronary steal, and most commonly occurs in individuals with extensive coronary artery disease.

A 23-year-old woman comes to the emergency department with vaginal bleeding and lower abdominal cramps. Her last menstrual period was 6 weeks ago, and a home pregnancy test was positive. She has no medical history and does not use tobacco or alcohol. Pelvic ultrasonography reveals a gestational sac in the left fallopian tube without evidence of rupture. A single dose of methotrexate is administered to treat her condition. Which of the following substances will most likely accumulate in embryonic tissues as a result of treatment? A. Dihydrofolate polyglutamate(50%) B. Folinic acid(6%) C. Para-aminobenzoic acid(8%) D. Tetrahydrofolate(29%) E. Thymidylic acid(5%)

Methotrexate (MTX), a folate antagonist, is the drug of choice for non-surgical treatment of an early, unruptured ectopic pregnancy. Folic acid is a required precursor to nucleic acid synthesis. Barriers to its metabolism are especially effective in hindering the growth of rapidly proliferating cells such as in a developing embryo or certain types of cancer. Normally, folic acid is reduced to dihydrofolate (DHF) by dihydrofolate reductase and then reduced further to tetrahydrofolate (THF), an integral precursor of DNA synthesis. MTX is structurally similar to folic acid and competitively and irreversibly inhibits DHF reductase. DHF reductase inhibition prevents the formation of THF (Choice D). After entering the target cell, MTX undergoes polyglutamation, which prevents the movement of MTX out of the cell, resulting in intracellular accumulation of MTX for later use. Similarly, folate and recycled DHF are stored within the cell via polyglutamation. Because MTX inhibits DHF reductase, folic acid and DHF polyglutamate will accumulate in cells.

A clinical trial is being conducted to evaluate the safety and efficacy of a novel therapy to treat refractory Crohn disease. The medication is a monoclonal antibody against the α4β7 integrin, which inhibits migration of T-lymphocytes into the intestinal parenchyma and produces a gut-selective anti-inflammatory effect. Patients who have active, moderate to severe Crohn disease and have failed conventional therapy are enrolled in the study. Many of these patients have renal or hepatic dysfunction, and some take other medications that affect cytochrome P450 enzymes. Which of the following is the most appropriate dose adjustment in this patient population to decrease drug toxicity? A.Higher dose in patients taking cytochrome P450 inducers B.Lower dose in patients with decreased glomerular filtration C.Lower dose in patients with hepatocellular dysfunction D.Lower dose in patients with renal tubular dysfunction E.No dose adjustment necessary

Monoclonal antibodies (mAbs) are used to treat a growing variety of malignant (eg, leukemia/lymphoma, melanoma) and autoimmune diseases (eg, Crohn disease, rheumatoid arthritis). The therapeutic effect of mAbs is achieved by binding to their target antigen in the plasma or on the cell surface and blocking deleterious receptor interactions or triggering a cytotoxic immune response against abnormal cells. Because of their large molecular size, mAbs cannot be administered orally and must be given via intravenous or subcutaneous/intramuscular routes. Unlike most other drugs, mAbs are not eliminated by hepatic or renal clearance, but are instead removed from the body in 2 primary ways: Target-mediated drug clearance: mAbs directed against cell surface antigens undergo internalization (receptor-mediated endocytosis) upon binding to their targets, removing them from the circulation Nonspecific clearance: Immunoglobulins are constitutively taken up by reticuloendothelial macrophages (via binding to Fc receptors) and vascular endothelial cells (via pinocytosis) Once internalized, immunoglobulins are catabolized into amino acids within lysosomes.

Biologists investigating the morphologic changes associated with reversible cellular injury perform a procedure on anesthetized mice to assess the effects of transient hepatic ischemia. During the experiment, they clamp the hepatic artery and obtain liver biopsy samples at varying intervals. The samples are then examined by electron microscopy. Cells that are exposed to longer ischemic periods are found to have reduced numbers of ribosomes attached to the endoplasmic reticulum. This structural change is most likely to impair which of the following cellular functions? A. ATP production(3%) B. Drug detoxification(3%) C. Synthesis of cell membrane proteins(64%) D. Synthesis of cytosolic proteins(24%) E. Synthesis of steroid hormones(4%)

Ribosomes are cellular organelles that synthesize proteins. Each ribosome consists of 2 subunits. The small ribosomal 40S subunit is responsible for binding mRNA (the protein synthesis template) and tRNA (carries amino acids). The larger 60S subunit contains peptidyl transferase, the enzyme that catalyzes peptide bond formation between amino acids. All ribosomes begin protein translation in the cytoplasm, but some translocate to the rough endoplasmic reticulum (RER) during protein synthesis depending on the protein's target destination. Free ribosomes remain floating in the cytosol throughout protein synthesis. They are responsible for translating proteins found within the cytosol, nucleosol, peroxisome matrix, and nuclear-encoded mitochondrial proteins (Choice D). Attached ribosomes bind to the RER after protein translation begins. They synthesize most secretory proteins, the integral membrane proteins of the nucleus and cell membrane, and proteins within the ER, Golgi network, and lysosomes. The RER is particularly well developed in protein-secreting cells (eg, pancreatic and plasma cells). Ribosomes attach to the RER via the translocon, a protein complex containing ribophorins that bind the large 60S subunit.

A 33 year-old female is being followed by her neurologist for her multiple sclerosis. She was initially diagnosed with relapsing remitting subtype after an episode of visual disturbance and an episode of paralysis. In her study of autoimmune diseases she encounters the topic of lymphocyte development and comes across a question which she poses to her neurologist during a routine follow-up visit: During the process of T-lymphocyte maturation, T cell receptors of many lymphocytes demonstrate a very high-affinity interaction with MHC molecules expressed on thymic medullary epithelial and dendritic cells. What process do these lymphocytes undergo at this time? A. Affinity maturation(4%) B. Isotype switching(2%) C. Negative selection(63%) D. Positive selection(28%) E. TCR DNA rearrangement(1%)

T-lymphocytes, or thymocytes, are initially produced in the bone marrow, but they migrate from that location to mature during the first trimester of gestation in the thymus. In the thymus, the processes of T-cell receptor (TCR) gene rearrangement, positive selection, negative selection, and expression of extracellular membrane markers and co-stimulatory molecules occurs. Pro-T cells arrive at the thymus as "double negative" cells - indicating that they lack both CD4 and CD8 antigens and begin their differentiation in the subcapsular zone. Next, the process of TCR beta gene rearrangement occurs with simultaneous expression of both CD4 and CD8. Once in thymic cortex, the alpha genes rearrange to produce a functional alpha-beta TCR. Subsequently, the processes of positive and negative selection occur with cells that fail either of these tests being eliminated by apoptosis. Positive selection is the process by which only T cells expressing a TCR that is able to bind self MHC are allowed to survive. Those cells expressing a TCR that is not specific for self MHC are signaled for elimination by apoptosis. This process occurs after TCR DNA rearrangement and prior to the process of negative selection. It occurs in the thymic cortex and involves interaction of T cells with thymic cortical epithelial cells expressing self MHC(Choice D). Positive selection is responsible for development of a T cell repertoire that can recognize self. Negative selection occurs after positive selection and is the process by which T cells possessing TCRs that bind with high affinity to self antigen or self MHC class I or II are eliminated by apoptosis. Negative selection occurs in the thymic medulla and involves interaction of the developing T cells with thymic medullary epithelial and dendritic cells. This process serves to eliminate T cells that may be overly autoreactive against self antigens and therefore may play a role in autoimmunity if not destroyed. This results in a population of T cells that have only an appropriately low affinity for self MHC molecules.

A 32-year-old woman comes to the clinic due to 6 weeks of bloating and nonbloody diarrhea. She describes having multiple watery bowel movements a day without visible blood. She underwent gastric bypass surgery for morbid obesity 3 years ago. The patient has no other medical conditions. She is a lifelong nonsmoker and does not drink alcohol. Her vital signs are within normal limits. Physical examination is unremarkable. A jejunal aspirate shows bacterial count of >105 organisms/mL, consistent with small intestinal bacterial overgrowth. Which of the following serum substance levels is most likely to be increased in this patient? A.Cobalamin B.Folate C.Iron D.Vitamin D2 E.Zinc

The normal small intestine is colonized with facultative anaerobes, lactobacilli, enterococci, and gram-positive aerobes. Enteric bacteria can produce vitamins (eg, vitamin K, folate), inhibit proliferation of surrounding pathogenic bacteria, and digest unabsorbed dietary sugars and convert them to fatty acids. A Roux-en-Y gastric bypass surgery typically creates a small gastric pouch, which is removed from the remainder of the stomach and attached to the jejunum via a gastrojejunal anastomosis. The larger bypassed portion of the stomach and duodenum are reattached to the jejunum distally. This results in a closed-ended gastroduodenal limb, in which bacteria can proliferate and ferment any food that may be diverted into this segment. Small intestinal bacterial overgrowth (SIBO) is characterized by overproduction of vitamin K and folate, associated with nausea, bloating, abdominal discomfort, and malabsorption.

A 25-year-old woman comes to the office due to a 4-week history of a pruritic skin rash. The patient has no chronic medical conditions and takes no medication. Vital signs are normal. On examination, there are pink papules distributed symmetrically over the anterior surfaces of the shins and ankles. Shave biopsy reveals prominent hyperkeratosis with a thickened granular layer. In addition, there is a bandlike infiltrate of mononuclear cells (predominantly lymphocytes) in the subjacent superficial dermis that involves the overlying epidermis. The rete ridges have a sharpened, sawtooth appearance. The basal layer of the epidermis is degenerated, with scattered colloid bodies. Which of the following is the most likely diagnosis? A. Atopic dermatitis(4%) B. Contact dermatitis(5%) C. Erythema nodosum(11%) D. Lichen planus(49%) E. Psoriasis(29%) Incorrect

This patient has a pruritic rash consistent with lichen planus (LP), an immunologically mediated skin disorder primarily affecting middle-aged adults. The skin lesions typically occur symmetrically on the flexural surfaces of the wrists and ankles but can also involve the nails, oral mucous membranes, and genitalia. The characteristic skin lesions are described by the 5 Ps: pruritic, purple/pink, polygonal papules and plaques and can form along lines of minor trauma (Koebner phenomenon). Chronic lesions often show white, lacy markings known as Wickham striae. Although the etiology of LP is unknown, it is characterized by a (CD8+) T cell-mediated response to the cells along the junction of the dermis and epidermis, resulting in the following characteristic histologic findings: Chronic hyperkeratosis (thickening of stratum corneum) Lymphocytic infiltrates at the dermoepidermal junction (interface dermatitis) Scattered eosinophilic, colloid (Civatte) bodies in the papillary dermis (apoptotic keratinocytes) Thickened stratum granulosum with sawtooth rete ridges

A newborn girl is evaluated in the neonatal intensive care unit for difficulty breathing. The patient was born at term via spontaneous vaginal delivery. Since birth, she has had difficulty breathing with loud snoring sounds and intermittent oxygen desaturations. She has been unable to breastfeed due to her breathing problems. Breathing improves significantly when the patient is placed in a prone position. On examination, there is a small mandible, posteriorly displaced tongue, and U-shaped cleft palate. The abnormalities described represent an example of which of the following? A. Association(6%) B. Disruption(15%) C. Imprinting(1%) D. Sequence(35%) E. Syndrome(41%) Incorrect

This patient has a small mandible, posteriorly displaced tongue, and U-shaped cleft palate, which is consistent with Pierre Robin sequence. A sequence occurs when a single developmental defect causes a cascade of additional malformations. In Pierre Robin sequence, hypoplasia of the mandibular prominence leads to micrognathia. Severe micrognathia causes posterior displacement of the tongue (glossoptosis), which blocks fusion of the palatine shelves, resulting in a cleft palate that is characteristically U-shaped. Difficulty breathing occurs because the tongue prolapses into the posterior oropharynx, blocking airflow. Breathing improves when the patient is in a prone position because gravity pulls the tongue anteriorly, opening the airway.

A 53-year-old man comes to the office due to difficulty breathing and increasing fatigue. He has shortness of breath at night and has been sleeping in a recliner to help relieve his dyspnea. Medical history is significant for hypertension and hyperlipidemia. Two months ago, the patient suffered a myocardial infarction that was not revascularized due to a delay in seeking treatment. He quit smoking afterward, but had previously smoked a pack of cigarettes daily for 25 years. On cardiac auscultation, a low-frequency diastolic heart sound is heard shortly after the second heart sound when the patient lies in the left lateral decubitus position. Cardiac imaging shows hypokinesis of the left ventricular free wall. Which of the following is most likely to accentuate this patient's abnormal auscultation finding? A.Amyl nitrite inhalation B.Furosemide injection C.Having the patient stand up D.Listening at end expiration E.Phase 2 of the Valsalva maneuver

This patient has decompensated heart failure (fatigue, dyspnea) with an audible third heart sound (S3), a low-frequency sound that occurs shortly after S2 during diastole. S3 is caused by a sudden limitation of ventricular movement during rapid passive ventricular filling. It can be a normal finding in healthy children and young adults. However, the presence of S3 in patients age >40 is considered abnormal and suggests ventricular volume overload or enlargement. It is often heard with aortic/mitral regurgitation or systolic heart failure (eg, dilated or ischemic cardiomyopathy). In this case, imaging is consistent with left ventricular systolic dysfunction secondary to the patient's recent myocardial infarction. Left ventricular gallops (S3 and/or S4) are best heard with the bell of the stethoscope over the cardiac apex while the patient is in the left lateral decubitus position. Listening at end expiration makes the sound even more audible by decreasing lung volume and bringing the heart closer to the chest wall.

A 14-year-old boy experiences severe, prolonged bleeding following a tooth extraction. He also has a history of multiple episodes of painful joint swelling following minor trauma. His parents have no bleeding problems. Evaluation shows that the patient has an inherited disorder and that one of his parents is a genetic carrier. His older sister, who does not have this condition, is pregnant. She does not know the sex of her child. She asks about the risk that her child will be affected. Which of the following is the best estimate that this child will have the disease? A.Near0 B.1/2 C.1/4 D.1/8 E.1/16 F.1/32

This patient is a boy with excessive bleeding and hemarthroses, suggesting a diagnosis of hemophilia A or B. Both diseases are X-linked recessive coagulation factor deficiencies. The probability that his sister will give birth to an affected child can be calculated by multiplying the following probabilities: The probability (p1) that the sister is a carrier = 0.5. The patient's father does not carry the mutation on his X chromosome because he would be affected by the disease if he did. That means the mother carries the mutation on 1 of her 2 X chromosomes. This gives the daughter a 50% chance of having inherited the mutated X chromosome and therefore being a carrier. The probability (p2) that the offspring of a female carrier will inherit the X chromosome with the hemophilia gene = 0.5. Assuming the daughter is a carrier, the probability of passing on the mutant allele is 50% as only 1 of her 2 X chromosomes is passed to her offspring. The probability (p3) that his sister will have a boy = 0.5. If the sister's child is female, the child could be a carrier of the disease but would not be affected by it. If a male child inherits the mutated X chromosome, he will have the disease. The probability that the sister will have an affected son is the probability that all 3 of the above events will take place (ie, the product of their individual probabilities): p1 x p2 x p3 = 1/2 x 1/2 x 1/2 = 1/8.

A 12-year-old boy is brought to the office due to gait instability and pruritic skin rash for the past several weeks. His mother reports that he has also been irritable and had loose stools during this time. The patient's childhood development has been unremarkable except for several episodes of similar skin rash that resolved spontaneously. Examination shows scaly, erythematous skin lesions in sun-exposed areas and cerebellar ataxia. Laboratory evaluation shows increased levels of neutral amino acids in the urine. This patient's symptoms would most likely respond to which of the following supplements? A.Ascorbate B.Folic acid C.Niacin D.Pyridoxine E.Riboflavin F.Thiamine G.Tocopherol

This patient likely has Hartnup disease, an autosomal recessive metabolic disorder caused by inactivating mutations affecting the neutral amino acid transporter. This results in impaired transport of neutral amino acids, particularly tryptophan, in the small intestine and proximal tubule of the kidney. Tryptophan is an essential amino acid and a precursor for niacin, serotonin, and melatonin. Conversion of tryptophan to niacin is responsible for the generation of up to half of the nicotinamide adenine dinucleotide (NAD+) required for redox reactions; the clinical manifestations of Hartnup disease are primarily due to niacin deficiency. Patients present with intermittent attacks of pellagra-like skin eruptions (development of a red, rough rash following sun exposure) and cerebellar ataxia in early childhood that become less severe with increasing age. The diagnosis is confirmed by detecting excessive amounts of neutral amino acids (alanine, serine, threonine, valine, leucine, isoleucine, phenylalanine, tyrosine, and tryptophan) in the urine (neutral aminoaciduria). A high-protein diet along with daily niacin or nicotinamide supplementation generally results in significant symptom improvement.

A 19-year-old man comes to the office with an ulcer on his penis. The patient first noticed the lesion 3 days ago. His temperature is 37.1 C (98.8 F). Physical examination shows an indurated and painless ulcer near the glans penis, with no surrounding erythema and no inguinal lymphadenopathy. The patient is a college student. He has no significant past medical history and takes no medications. He has no known drug allergies. The first-line treatment for this patient has structural similarities with which of the following? A.D-alanine-D-alanine B.D-glutamic acid-D-glutamic acid C.Folic acid D.L-alanine-D-glutamine E.N-acetylglucosamine F.N-acetylmuramic acid

This patient likely has a chancre due to primary syphilis, a sexually transmitted infection caused by the spirochete Treponema pallidum. Spirochetes are universally sensitive to penicillin because they are all gram-negative organisms with a peptidoglycan cell wall. Penicillins work by covalently binding to and inhibiting transpeptidase. This enzyme catalyzes the final cross-linking step in peptidoglycan cell wall formation: the joining of the amino acid in the third position of a peptidoglycan molecule to the terminal D-alanine-D-alanine of another peptidoglycan molecule. The structural similarity of penicillins to D-alanine-D-alanine likely facilitates their binding to transpeptidase. When transpeptidase is inhibited, cell wall synthesis ceases and cell wall degradation by bacterial autolysins proceeds unchecked. The weakened cell wall integrity causes osmotic lysis of the bacterium. Vancomycin, a glycopeptide antibiotic, directly binds to the D-alanine-D-alanine residues, preventing the incorporation of new subunits into the cell wall. Vancomycin acts at an earlier stage of cell wall formation than do the penicillins.

A 62-year-old man comes to the emergency department due to a 2-week history of worsening shortness of breath. The patient reports feeling dyspneic while lying flat and has been sleeping upright in his recliner. Temperature is 36.7 C (98.1 F), blood pressure is 110/70 mm Hg, pulse is 88/min, and respirations are 20/min. On physical examination, jugular venous distension is present. Heart sounds are normal. Decreased breath sounds and dullness to percussion can be heard at the bilateral bases. Pitting edema is present in the bilateral lower extremities. Chest x-ray shows cardiomegaly and bilateral pleural effusions. Serum protein is 6 g/dL and serum lactate dehydrogenase (LDH) is 60 U/L. Which of the following sets of pleural fluid findings is most likely to be seen in this patient? Protein(g/dL) LDH(U/L) Glucose(mg/dL) Total nucleated cell count(per mm3) A.P= 2, LDH 30, GLU 90, TOTAL NUCLEATED CEL 500 B.2, 90, 10, 10,000 C.2, 90, 90, 500 D.4, 90, 10, 10,000 E.4, 30, 90, 500

This patient with dyspnea, orthopnea, jugular venous distension, and lower extremity swelling likely has an acute heart failure exacerbation. Heart failure commonly leads to pleural effusion due to poor forward blood flow from the left ventricle and a subsequent increase in pulmonary venous and pulmonary capillary hydrostatic pressure. Pleural effusions that result from such pressure changes are typically transudative, whereas those that result from an inflammatory increase in vascular membrane permeability are typically exudative. Because transudative and exudative pleural effusions have differing underlying causes, they also tend to differ in chemical makeup. The Light criteria are used to differentiate the 2 types via analysis of the total protein and lactate dehydrogenase (LDH) levels. Exudative effusions have a high pleural fluid/serum ratio of these proteins due to increased capillary permeability, while transudative effusions are associated with a low pleural fluid/serum ratio. Glucose levels and leukocyte counts can suggest the degree of inflammation in a pleural effusion. Because leukocytes metabolize glucose, highly inflammatory effusions typically have low glucose levels. Transudative effusions are not inflammatory and almost always have low nucleated cell counts and normal (or high) glucose levels.

A 40-year-old woman is brought to the emergency department by her roommate due to significant left leg weakness. The symptom began 3 days ago after her father had a heart attack. There is no personal or family history of neurological disease; surgical history includes liposuction of the thighs and varicose vein removal. She does not use tobacco, alcohol, or illicit drugs. Temperature is 36.7 C (98.1 F), blood pressure is 123/81 mm Hg, pulse is 62/min, and respirations are 14/min. Physical examination reveals symmetric 2+ deep tendon reflexes as well as normal muscle bulk and tone bilaterally in the upper and lower extremities. Laboratory testing and neurologic imaging reveal no abnormalities. Which of the following is the most likely diagnosis? A. Body dysmorphic disorder(0%) B. Conversion disorder(66%) C. Factitious disorder(3%) D. Illness anxiety disorder(4%) E. Malingering(1%) F. Somatic symptom disorder(22%)

This patient's acute onset of muscular weakness following a stressful incident, normal neurological examination, and negative workup are suggestive of conversion disorder (functional neurologic symptom disorder). Conversion disorder is characterized by symptoms or deficits of voluntary motor and/or sensory function that are incompatible with any recognized neurological condition and cannot be explained by another medical or mental disorder. Common symptoms of conversion disorder may include weakness, paralysis, gait disturbance, blindness, diplopia, aphonia, anesthesia, or nonepileptic seizures (also called psychogenic seizures). The phenomenon of la belle indifférence (ie, incongruous lack of concern about symptoms) has been associated with conversion disorder but is not pathognomonic, occurs at equal frequency in true neurological disorders, and should not be used to make the diagnosis. In fact, patients with conversion disorder may be very distressed by their symptoms.

A 25-year-old man comes to the office due to shortness of breath. He states that even moderate exertion forces him to stop to catch his breath. The patient has no family history of sudden cardiac death. He does not use tobacco or illicit drugs. On examination, the patient has a faint systolic murmur at the left sternal border in the supine position that increases to 3/6 in intensity with Valsalva maneuver. Echocardiogram shows interventricular septal hypertrophy and increased left ventricular mass. The patient is started on high-dose beta blocker therapy. On follow-up 4 weeks later, he reports significant improvement in his shortness of breath. Physical examination reveals no murmurs supine or with Valsalva. Which of the following best accounts for improvement of this patient's symptoms? A. Dilation of epicardial coronary arteries(2%) B. Increase in left ventricular outflow tract flow velocity(16%) C. Reduction in left ventricular contractility(63%) D. Reduction in left ventricular mass(4%) E. Reduction in left ventricular preload(13%) Incorrect

This patient's echocardiogram shows increased left ventricular (LV) mass with predominant interventricular septal hypertrophy, which are classic findings for hypertrophic cardiomyopathy (HCM). Many patients with HCM have LV outflow tract obstruction that can lead to symptoms of poor cardiac output (eg, dyspnea, syncope). The outflow tract obstruction worsens with decreased LV blood volume (as can occur during exercise due to decreased LV filling time and increased LV contractility); on physical examination, this effect is evidenced by a systolic murmur that increases in intensity with maneuvers that decrease LV blood volume (eg, Valsalva strain phase, abrupt standing). In patients with HCM and symptomatic LV outflow tract obstruction, beta blockers (eg, metoprolol, nadolol) improve symptoms mostly via an increase in LV volume that reduces LV outflow tract obstruction and improves cardiac output. This occurs via the following 2 mechanisms: Beta blockers reduce heart rate, which lengthens diastolic filling time and increases the amount of blood that can enter the heart during each beat (increasing end-diastolic volume) Beta blockers also reduce LV contractility, which reduces the amount of blood ejected during systole (increasing end-systolic volume)

A 28-year-old man is brought to the emergency department by his roommate, who is concerned about his change in behavior over the past 2 weeks. The roommate describes the patient as "a regular guy who is usually very responsible." Last week, the patient abruptly quit his job as a computer programmer and started placing large bets on an online gambling site because he was "sure to make millions." The roommate says that the patient has been staying up most nights scribbling notes for his autobiography on small scraps of paper. The patient says, "My new mission is to spread understanding." He denies any alcohol or drug use, which his roommate affirms. This patient is most likely to exhibit which of the following additional findings? A.Flat affect B.Low self-esteem C.Poor hygiene D.Pressured speech E.Psychomotor slowing F.Social withdrawal G.Thought blocking

This patient's grandiose ideas about his special mission, impulsive risk-taking behavior, and decreased need for sleep lasting ≥1 week are suggestive of a manic episode consistent with bipolar I disorder. Other features of mania include elevated/euphoric/irritable mood, increased energy, and hyperactivity. Increased production, volume, and rate of speech (ie, pressured speech) and a sense that one's thoughts are moving very quickly (ie, racing thoughts) are also common. Diagnosis of bipolar I disorder requires ≥1 manic episodes, although most bipolar patients will experience both major depressive and manic episodes in their lifetime. Manic episodes can occur with or without psychotic features (eg, delusions, hallucinations). Pharmacotherapy for acute mania includes mood stabilizers such as lithium and valproate and second-generation antipsychotics (eg, risperidone).

A 65-year-old man is brought to the emergency department with new-onset confusion, suprapubic discomfort, and lack of urine output. His past medical history is significant for benign prostatic hyperplasia, hypertension, hyperlipidemia, and type 2 diabetes mellitus complicated by neuropathy. The patient also has a history of chronic insomnia and has been treated with several medications with little benefit. He does not know his current medications. On examination, the patient is afebrile, confused, and oriented only to self. Suprapubic fullness is present, but abdominal examination is otherwise unremarkable. A urinary catheter is placed and immediately drains 1000 mL of urine. Which of the following medications most likely has contributed to this patient's current condition? A. Amitriptyline(57%) B. Atorvastatin(1%) C. Duloxetine(8%) D. Finasteride(6%) E. Gabapentin(10%) F. Metformin(2%) G. Tamsulosin(14%)

Tricyclic antidepressants (TCAs), such as amitriptyline, are occasionally used for insomnia or adjunctive pain management when other medications are not effective. However, TCAs have strong anticholinergic properties that can lead to significant adverse effects such as confusion, constipation, and acute urinary retention. Elderly patients are at increased risk for side effects due to comorbid conditions (eg, dementia, benign prostatic hyperplasia), decreased hepatic and renal clearance of medications, and a higher burden of concurrent medications that can interact with TCA's and contribute to adverse effects. As a result, TCAs are relatively contraindicated in elderly patients.

A 42-year-old man comes to the office due to numbness and tingling in both legs and difficulty walking for the past several months. He has also noticed that he tires more easily with physical activity. His temperature is 36.8 C (98 F), blood pressure is 122/86 mm Hg, pulse is 76 /min, and respirations are 14/min. Physical examination shows conjunctival pallor and loss of vibration and position sensation in the bilateral lower extremities with associated gait ataxia. The remainder of the examination is within normal limits. Which of the following findings is most likely to be present upon further questioning of the patient? A.Ongoing treatment for latent tuberculosis(5%) B.Strict vegan diet for the past 6 years(65%) C.Total gastrectomy 8 months ago(26%) D.Use of an anti-epileptic drug for the past 3 months(2%) E.Working indoors most of the time(0%) Incorrect

Vitamin B12 (cobalamin) is obtained solely from the diet, specifically from animal products such as meat, dairy, and fish. Vitamin B12 cannot be obtained from plant products, placing strict vegans at risk for dietary deficiency. The body is capable of storing about 1,000 times the daily requirement; therefore, vitamin B12 deficiency develops only after the complete absence of intake for 4-5 years. The development of anemia as well as severe and potentially irreversible neurologic damage can result from vitamin B12 deficiency. Red blood cell synthesis relies on vitamin B12-dependent recycling of folate; deficiency of either vitamin results in a megaloblastic anemia. Neurologic symptoms vary in severity, but patients typically present with both motor and sensory deficits. Impaired myelin synthesis, specifically in the dorsal and lateral columns, results in subacute combined degeneration of the spinal cord. This condition is progressive and begins as a symmetrical neuropathy consisting of paresthesias and weakness. Ongoing deficiency leads to loss of vibration and position sensation with development of an ataxic gait. The longer the deficiency is untreated, the less likely it can be reversed.

A 7-year-old boy is brought to the emergency department due to fever, headache, vomiting, and confusion. He has no prior medical conditions and is up to date with immunizations. The patient has not traveled recently, and his parents report no exposure to individuals with similar symptoms or ingestion of anything out of the ordinary. They have recently noticed several dead birds around their neighborhood. Temperature is 39 C (102.2 F). Physical examination shows neck rigidity and right lower extremity weakness with fasciculations. Cerebrospinal fluid analysis shows a leukocyte count of 200/mm3 with 20% neutrophils and 80% lymphocytes. Which of the following is the most likely pathogen responsible for this patient's condition? A. Cryptococcus neoformans(27%) B. Hemophilus influenzae(2%) C. Herpes simplex virus(2%) D. Nonpolio enterovirus(19%) E. Polio virus(6%) F. Streptococcus pneumoniae(1%) G. West Nile virus(40%)

West Nile virus is an enveloped RNA virus that is found in warm climates worldwide. The virus replicates extensively within birds and is passed to mosquitos (Culex species) during blood feeding. Viral accumulation in mosquito salivary glands can lead to human transmission. Most infected patients develop low-level viremia that is neutralized within a week by the humoral immune response; these individuals typically remain asymptomatic. However, a minority of patients (particularly the young, elderly, or immunosuppressed) are unable to mount an effective immune response and develop significant clinical manifestations, including: West Nile fever - a self-limited, flu-like illness that is often associated with a maculopapular rash Neuroinvasive disease - the virus is neurotropic and often causes meningitis (eg, fever, headache, neck stiffness), encephalitis (eg, confusion, tremors, seizures), and/or flaccid paralysis (eg, asymmetric weakness of limbs, fasciculations). Cerebrospinal fluid usually shows a lymphocytic pleocytosis. Most patients recover completely with supportive care, but some with neuroinvasive disease have long-term neurologic sequelae or die.

A 57-year-old Caucasian male with severe pyelonephritis is admitted to the hospital. His past medical history is significant for diabetes, hypertension, and two episodes of transient ischemic attacks. His serum creatinine level is 3.2 mg/dL; therefore, he needs to be started on an antibiotic that depends mainly on non-renal clearance. Which of the following characteristics should the antibiotic also have if hepatic metabolism and clearance is desired? A.Low volume of distribution B.Poor oral absorption C.High lipophilicity D.Low rate of redistribution E.Poor penetration into the CNS

While the kidney is the primary site of elimination of most drugs, the liver is the main site of biotransformation of these agents in preparation for elimination. Drugs that are more lipophilic (high Vd, good penetration into CNS) are preferentially processed by the liver into more polar compounds for easier elimination in the bile and urine. Liver disease (e.g., cirrhosis) or the concomitant use of other drugs may limit or enhance the clearance of drugs metabolized in the liver.

A 32-year-old woman comes to the outpatient clinic due to shortness of breath for the past 6 months. The patient says she can walk only 3-4 blocks on level ground before feeling out of breath. She also has an occasional dry cough at night. The patient denies any chest pain, palpitations, syncope, weight loss, or lower extremity swelling. She immigrated from Guatemala 8 years ago. Medical history is unremarkable. On physical examination, the patient appears comfortable. Blood pressure is 128/75 mm Hg and pulse is 78/min and regular. Peripheral pulses are full and symmetric. The lungs are clear on auscultation. S1 is loud. With the patient positioned on her left side and breath held at the end of expiration, a diastolic murmur is heard at mid- and end diastole. Otherwise, examination is unremarkable. Which of the following are the most likely left ventricular hemodynamic findings at rest in this patient? Diastolic pressure, afterload and contractility?

all stay normal This patient's mid-to-late diastolic murmur is consistent with mitral stenosis (MS). MS usually occurs due to underlying rheumatic heart disease, which in the United States is most commonly seen in patients who emigrated from Latin America, Africa, or Asia. MS can lead to heart failure due to restriction of left ventricular (LV) diastolic filling. Patients usually experience exertional dyspnea as the first symptom, which results from an inability to increase cardiac output to meet exertional demands. As MS worsens, elevated left atrial pressure can lead to orthopnea, paroxysmal nocturnal dyspnea, productive cough/hemoptysis, and eventual right-sided heart failure with resulting jugular venous distension and lower extremity edema. This patient's presentation with exertional dyspnea in the absence of volume overload (ie, dry cough, no pulmonary crackles, no peripheral edema) suggests relatively early, mild-to-moderate disease. A loud S1 (caused by the stiff leaflets snapping shut) is further suggestive of more mild disease, as the intensity of S1 diminishes with disease progression due to heavy calcification that limits valve motion. Although severe MS can cause reductions in LV diastolic pressure and afterload (due to reduced ventricular filling), these parameters typically remain normal until later in the disease course (especially at rest). LV contractility is also normal at rest in early disease; however, with severe MS the reduction in cardiac output stimulates increased sympathetic activity to increase LV contractility.

A 57-year-old woman comes to the office due to left breast pain associated with fatigue, nausea, and unexplained weight loss. She is referred for mammography, which reveals a 3-cm dominant mass in the left breast. Biopsy confirms the mass to be adenocarcinoma, and subsequent studies find that the tumor is estrogen receptor-positive and human epidermal growth factor receptor 2- (HER2) negative. The patient experienced menopause at age 52, and her past medical history is otherwise unremarkable. Anastrozole treatment is initiated and leads to a substantial decrease in the size of the primary tumor. Which of the following is the best explanation for the therapeutic effect of this agent? A. Decreased androgen aromatization(70%) B. Decreased androgen synthesis(9%) C. Decreased follicular cell stimulation(3%) D. Impaired ligand-receptor interaction(14%) E. Impaired second-messenger action(2%)

anastroazole is a aromatase inhibitor Estrogens are synthesized primarily by the aromatization of androgens. Aromatase belongs to the P450 superfamily and is highly expressed in ovarian tissue (in which expression is gonadotropin dependent). Aromatase is also expressed in the adrenal cortex, subcutaneous fat, and breast. In the postmenopausal state, follicular atresia leads to reduced numbers of granulosa cells and decreased ovarian estrogen synthesis. However, the ovaries and adrenal glands continue to produce androgens in significant quantities, and extraovarian aromatase maintains a low level of estrogen in the peripheral circulation. Estrogen is the main hormone driving the growth and development of estrogen receptor (ER)-positive malignant breast tumors. Aromatase inhibitors (eg, anastrozole, letrozole, exemestane) reduce the synthesis of estrogen from androgens, suppressing estrogen levels in postmenopausal women and slowing progression of ER-positive tumors. They are less effective as monotherapy in premenopausal women, as ovarian aromatase is upregulated substantially in response to gonadotropins.

A 16-year-old boy is brought to the office by his parents due to tremors and difficulty walking. Over the past year, his school performance has declined, and he has been moodier and more impulsive. His parents also note that he has been sleeping more than usual and has had a poor appetite for the past 2 months. Approximately 3 weeks ago, the patient developed a tremor in both hands that is most prominent when he reaches for an object or tries to write. The tremor has worsened significantly over the past week, and he has developed an unsteady, broad-based gait. Laboratory studies reveal elevated serum transaminases and 24-hour urinary copper excretion. The patient is prescribed penicillamine therapy. This medication primarily works through which of the following mechanisms? A.Decreasing intestinal absorption of copper B.Increasing urinary excretion of copper C.Repletion of intracellular glutathione D.Stimulation of defective gene expression E.Stimulation of canalicular transport proteins

b A -> zinc does this and can be used with penicillamine This patient has Wilson disease, an autosomal recessive disorder characterized by excessive copper deposition in the liver, basal ganglia, and cornea. Copper is an essential nutrient that is absorbed in the intestines, metabolized by hepatocytes, and excreted in the bile. Pathogenesis of Wilson disease involves impaired hepatocellular copper transport, which results in decreased biliary excretion of copper. The resultant increase in intracellular copper causes oxidative stress and apoptosis of hepatocytes, which leads to liver dysfunction (eg, elevated transaminases, liver failure). Excess copper is then released into the bloodstream and deposited into extrahepatic tissues, causing neurologic (eg, tremor, ataxia) and psychiatric symptoms (eg, impulsivity, depressive symptoms). Although renal excretion of copper in healthy patients is extremely low, the relative increase in serum copper in Wilson disease results in increased urinary excretion of copper, as seen in this patient. First-line treatment of Wilson disease is with penicillamine, a copper chelating agent. Penicillamine binds free copper as well as reduces copper bound to other proteins. This reduction preferentially increases affinity of copper to the chelating agent. The chelator-copper complex is soluble, thereby increasing urinary copper excretion.

A 58-year-old man comes to the emergency department due to generalized weakness, anorexia, and nausea for the past several weeks. He also reports lower extremity swelling but has had no dyspnea or chest pain. The patient was diagnosed with hypertension several years ago but did not follow up and takes no medications. Blood pressure is 182/100 mm Hg and pulse is 84/min. Physical examination shows pitting edema of the bilateral lower extremities. Laboratory studies reveal elevated serum creatinine and blood urea nitrogen levels. During evaluation of renal dysfunction, total urinary creatinine is measured over a 24-hour period, and creatinine clearance is calculated using the serum and urine creatinine concentrations and urinary volume. Compared to the calculated creatinine clearance, this patient's true glomerular filtration rate is most likely to be: A.20% higher B.20% lower C.90% higher D.90% lower E.Equal

b Creatinine is released from muscle at a relatively constant rate and is neither metabolized nor reabsorbed by the kidney. However, in addition to passive filtration, a portion of creatinine is actively secreted by the proximal tubules. Therefore, uncorrected creatinine clearance overestimates the GFR by approximately 10%-20%. Creatinine clearance has further limitations in patients with low muscle mass (eg, malnutrition, lower extremity amputation) or high- or low-protein diets.

In an experiment, cultured fibroblasts are mechanically lysed, and the membrane lipids and cellular proteins are chemically removed to isolate nucleic acids. The cellular extract containing the purified nucleic acids is incubated along with short sequences of repeated deoxythymidine residues fixed to latex beads. The solution is washed several times to remove unbound molecules. Which of the following types of nucleic acid is most likely to bind the strongest to the latex beads in this experiment? A.Aminoacyl-tRNA B.Mature mRNA C.Promoter regions of DNA D.Ribosomal RNA E.Splice sites of pre-mRNA F.Telomere regions of chromosomes

b In the experiment described above, the poly-A tail on mature mRNA is most likely to bind the latex beads because the adenine residues in this tail would form complementary base pairs with the repeated deoxythymine residues fixed to the beads. Mature mRNA refers to mRNA that has been processed and is ready for nuclear export and translation into protein. mRNA processing (post-transcriptional modification) involves the following steps: 5' capping: A 7-methyl-guanosine cap is added to the 5' end of the mRNA. Polyadenylation: A poly-A tail (chain of adenine residues) is added to most eukaryotic mRNA molecules by poly-A polymerase. Poly-A tails are not transcribed from the DNA template. Instead, a consensus sequence (AAUAAA) located near the 3' end of the RNA molecule directs the addition of the poly-A tail. This tail protects the mRNA from degradation within the cytoplasm after it exits the nucleus. Splicing: The initial mRNA transcript (pre-mRNA) contains sequences from coding and noncoding regions of DNA, known as exons and introns, respectively. Spliceosomes (complexes of small nuclear ribonucleoproteins [snRNPs] and other proteins) remove introns containing GU at the 5' splice site and AG at the 3' splice site (Choice E)

A 12-year-old Caucasian male with history of seizure disorder experiences several stroke-like episodes with residual neurological deficit. He also suffers from muscle weakness. Blood tests show increased serum lactate levels both post-exercise and at rest. This patient's condition is known to be maternally inherited. This patient's sister is also affected by the same disorder, but she displays very few symptoms. Which of the following is the most likely explanation for the variability in clinical presentation between the patient and his sister? A.Genetic imprinting B.Heteroplasmy C.Anticipation D.Low expression variability E.Female sparing

b Mitochondria have a small amount of their own DNA, called mitochondrial DNA (mtDNA), which can have deletions and point mutations just like regular DNA. Mitochondrial disorders (mutations in mtDNA) are unique in that they are exclusively inherited from one's mother. Recall that the ovum is relatively large and has many copies of mtDNA; whereas, the few copies of mtDNA present in sperm are lost during fertilization. Mitochondrial diseases affect both male and female offspring with equal frequency (100%), but there are variable degrees of severity. This variability occurs because, during mitosis, mitochondria are randomly distributed between daughter cells. As a result, some cells contain mitochondria with mostly damaged mtDNA, while some contain mostly normal mitochondrial genomes. This mixture of two types of genetic material is called heteroplasmy and is responsible for the clinical variability of mitochondrial diseases. The following mitochondrial syndromes are important: Leber hereditary optic neuropathy leads to bilateral vision loss. Myoclonic epilepsy with ragged-red fibers: myoclonic seizures and myopathy associated with exercise. Skeletal muscle biopsy shows irregularly shaped muscle fibers (ragged red fibers). Mitochondrial encephalomyopathy with lactic acidosis and stroke-like episodes (MELAS). The clinical presentation of MELAS is described in this vignette.

A 32-year-old man comes to the clinic for peptic ulcer disease follow-up. The patient has received several months of proton pump inhibitor therapy without significant improvement in his epigastric discomfort. He does not use nonsteroidal anti-inflammatory drugs, tobacco, or alcohol. Helicobacter pylori testing is negative. The patient undergoes a partial gastrectomy for refractory peptic ulcer disease. The pathologist receives the tissue and notes significant enlargement of the gastric rugal folds on gross examination. Microscopy of the gastric mucosa reveals parietal cell hyperplasia. Which of the following stimuli is the most likely cause of parietal cell proliferation in this patient? A.Acetylcholine B.Gastrin C.Secretin D.Serotonin E.Somatostatin F.Transforming growth factor alpha

b Parietal cells are located in the gastric glands of the fundus and body of the stomach. They can be spotted easily due to their eosinophilic cytoplasm on H&E stain. These cells have abundant mitochondria and an intracellular tubulovesicular system that allows them to secrete large quantities of gastric acid (HCl) and intrinsic factor. Parietal cells are influenced by a number of substances. Histamine, acetylcholine, and gastrin increase gastric acid secretion, but prostaglandins inhibit it. Gastrin not only stimulates HCl secretion, but it also has a trophic effect on parietal cells. In patients with Zollinger-Ellison syndrome (such as this patient), gastrin hypersecretion induces parietal cell hyperplasia, causing visible enlargement of gastric folds on endoscopy. The increased gastric acid secretion induced by excess gastrin also causes peptic ulcer disease, heartburn, and diarrhea.

A 38-year-old man comes to the emergency department because he has been vomiting blood. After appropriate resuscitation measures, he undergoes upper gastrointestinal endoscopy, which reveals a bleeding duodenal ulcer. During hospital day 2, the patient develops decreased urine output. Serum creatinine rises to 3.0 mg/dL from a baseline of 1.2 mg/dL. Renal biopsy shows patchy epithelial necrosis of the tubules, tubulorrhexis, and intratubular casts. On hospital day 8, urine output significantly increases and serum creatinine levels decline. Over the next few days, this patient is at highest risk for which of the following complications? A.Hyperphosphatemia B.Hypokalemia C.Metabolic acidosis D.Urinary protein loss E.Volume overload

b This patient developed acute renal failure after gastrointestinal hemorrhage; renal biopsy showing epithelial necrosis of the tubules, tubulorrhexis, and intratubular casts is consistent with acute tubular necrosis (ATN). ATN is characterized by tubular injury due to renal ischemia (eg, shock, hemorrhage) or direct cytotoxicity (eg, radiologic contrast agents, aminoglycosides). The clinical course of ATN can be broken into 3 stages. The initiation stage is marked by the inciting event (eg, hemorrhage, as in this patient) and the onset of tubular injury. If significant tubular damage occurs. the maintenance stage (oliguric stage) follows in 24-36 hours. During this stage, urine output decreases and patients may develop volume overload. Renal tubular dysfunction results in the characteristic low urinary osmolality (<350 mOsm/kg), high urinary sodium (>30 mEq/L), and high urinary fractional sodium excretion (>1%). In spite of the seemingly profound damage that occurs to nephrons in ATN, tubular epithelial cells have excellent regenerative capacity. If the patient survives the maintenance stage (by conservative management or dialysis), the recovery stage follows in 1-3 weeks. Glomerular filtration rate often improves before renal tubular function is restored, so patients can develop transient polyuria (sometimes >3 L/day) with significant electrolyte wasting because tubular resorptive capacity remains impaired. During this time, patients are at high risk of developing clinically significant electrolyte imbalances, particularly hypokalemia, which can be life-threatening. Serum concentrations of magnesium, phosphorus, and calcium may also be low. Most patients recover completely, depending on the magnitude of the initial injury.

A 55-year-old Caucasian male is found on colonoscopy to have a solitary mass in his sigmoid colon. Biopsy is consistent with colon cancer, and surgery is scheduled. Which of the following features would carry the worst prognosis in this patient? A.Presence of clinical symptoms B.Tumor penetration into the muscularis propria C.Poor differentiation of tumor cells D.High degree of tumor cell aneuploidy E.High number of mitotic figures

b Tumor stage is best determinant for the stage. The extent of tumor expansion is characterized by the stage of a tumor. The degree of tumor differentiation (from well-differentiated to anaplastic) is referred to as the grade. Tumor stage is the most important criteria for determining prognosis. If the tumor is confined to the mucosa (stage A), the patient has 90%+ chance of 5-year survival. If it involves the muscular layer, 5-year survival rate is 70-80%. Lymph node involvement (stage C) and distant metastasis (stage D) have poor prognosis.

A 26-year-old man comes to the emergency department due to chest pain, palpitations, shortness of breath, and sweating. He has been to the emergency department twice in the past month for similar symptoms. The patient has no other medical problems. He drinks 1 or 2 glasses of beer daily but does not use tobacco or illicit drugs. Family history is not significant. Blood pressure is 140/90 mm Hg, pulse is 96/min, and respirations are 20/min. ECG is normal. The patient is tremulous and says, "I feel like I'm going to die." Which of the following is the most appropriate next step in management? A.24-hour urine fractionated catecholamines and metanephrines B.Benzodiazepine administration C.Cardiac enzymes D.Holter monitor E.Initiation of selective serotonin reuptake inhibitor F.Urine toxicology

b dude has a panic attack

A 71-year-old man comes to the hospital due to 3 hours of persistent retrosternal chest pain and dyspnea. After prompt evaluation, he is diagnosed with acute myocardial infarction and undergoes cardiac catheterization. During cannulation of the right common femoral artery, the arterial wall is penetrated superior to the right inguinal ligament. Percutaneous coronary intervention is then performed, after which firm pressure is applied to achieve hemostasis. Soon after the procedure, the patient becomes cold, clammy, and hypotensive. Physical examination shows a 5cm region of ecchymosis surrounding the femoral puncture site. Internal bleeding is suspected. Which of the following is the most likely location of the blood collection? A.Pelvic cavity B.Retroperitoneal space C.Right paracolic gutter D.Subcutaneous tissue E.Thigh muscles

b qid:11764 vascular access during cardiac catheterization is typically obtained through either the common femoral artery or radial artery. The common femoral artery is the continuation of the external iliac artery as it crosses the inguinal ligament. Arterial puncture above the inguinal ligament increases the risk of retroperitoneal hemorrhage, as this portion of the vessel lies directly inferior to the peritoneum. Accidental puncture of the posterior wall can cause blood to track along the loose connective tissue surrounding the vessel and accumulate within the interfascial planes of the retroperitoneum. Bleeding in the retroperitoneal space cannot be controlled with external compression and can lead to life-threatening hemorrhage. Affected patients typically develop hemodynamic instability with significant hypotension, a drop in hemoglobin, and ipsilateral flank pain.

A 46-year-old woman comes to the emergency department due to progressive dyspnea over the last 2 days. The patient's exercise tolerance has decreased dramatically and she had to sleep in a sitting position last night. She has no history of cardiovascular disease. The patient does not use tobacco or alcohol. Family history is unremarkable. Blood pressure is 110/65 mm Hg and pulse is 105/min and regular. The apical impulse is hyperdynamic. Cardiac auscultation reveals a diminished S1 and an apical holosystolic murmur radiating to the axilla. Diffuse pulmonary crackles are heard bilaterally. There is no peripheral edema. ECG shows sinus tachycardia but is otherwise unremarkable. Which of the following would most likely increase the ratio of forward flow volume to regurgitant flow volume in this patient? A.Decreasing heart rate B.Decreasing systemic vascular resistance C.Decreasing venous return to the left ventricle D.Increasing left ventricular contractility E.Increasing venous return to the left ventricle

basically decrease afterload, because remmeber pressure determines flow and we always go from high to low so if you have decrease after load aka SVR then the flow will be more to the aorta and less towards right atrium This patient's dyspnea, orthopnea, and crackles on lung auscultation are consistent with decompensated left-sided heart failure; the apical holosystolic murmur radiating to the axilla suggests mitral regurgitation (MR) as a contributing factor. In patients with MR, some of the blood in the left ventricle is pumped forward through the aortic valve (forward stroke volume), and some is forced backward through the incompetent mitral valve (regurgitant stroke volume). The amount of blood flowing down each pathway is determined by the relative contribution of the resistance of each pathway to the total left ventricular afterload: Resistance to forward flow is primarily determined by the pressure in the aorta (systolic blood pressure) Resistance to regurgitant flow is determined by the mitral valve orifice size during systole and the degree of left atrial compliance. In chronic MR, the left atrium becomes more compliant and the lower left atrial pressures facilitate greater regurgitant flow. Left atrial pressure remains relatively constant over the short term, but aortic pressure can vary significantly with changes in systemic vascular resistance. A reduction in systemic vascular resistance leads to reduced systemic blood pressure and an increase in the ratio of forward to regurgitant blood flow. Pharmacologic vasodilators (eg, nitroprusside) therefore help to increase forward cardiac output and reduce pulmonary congestion in patients with MR

A 2-day-old girl is in the newborn nursery with persistent crying, tremors, tachypnea, sneezing, and diarrhea. She was born vaginally and had been breastfeeding well until several hours ago when she became tachypneic. Her mother has poorly controlled schizophrenia and did not receive prenatal care. The patient's mother also had a positive hepatitis C antibody test during postnatal laboratory testing. On physical examination, the girl has increased tone in all extremities. She is irritable during examination but quiets when swaddled. Chest radiograph shows normal lung fields. Which of the following is the most appropriate pharmacotherapy for treatment of the newborn's symptoms? A.Flumazenil B.Folic acid C.Methadone D.Naloxone E.Sodium bicarbonate F.Vitamin K

c

A 45-year-old woman comes to the office for progressive itchiness and fatigue. She also notes yellowing of the eyes and skin. Physical examination shows scleral icterus, multiple excoriations on both the upper and the lower extremities, and hepatomegaly. Laboratory results are as follows: Liver function studies Total bilirubin 5.3 mg/dL Alkaline phosphatase 982 U/L Aspartate aminotransferase (SGOT) 89 U/L Alanine aminotransferase (SGPT) 67 U/L Gamma-glutamyl transpeptidase 450 U/L Liver biopsy reveals dense lymphocytic infiltration of the portal triads, as well as granulomatous destruction of intralobular bile ducts. Which of the following is the most likely diagnosis? A.Acute hepatitis B B.Autoimmune hepatitis C.Primary biliary cholangitis D.Primary sclerosing cholangitis E.Sarcoidosis

c This patient with fatigue, pruritus, jaundice, and elevated alkaline phosphatase has liver biopsy results consistent with primary biliary cholangitis (PBC), a chronic autoimmune disorder that disproportionately affects middle-aged women. PBC is characterized by T-cell-mediated destruction of the small intralobular bile ducts, which is visualized histologically as a dense lymphocytic inflammation of portal tracts with granulomatous destruction of interlobular bile ducts (ie, florid duct lesions). PBC typically develops insidiously; fatigue and pruritus are often the presenting symptoms. Cholestasis results in a characteristic pattern of liver injury (ie, elevated alkaline phosphatase, elevated gamma-glutamyl transferase, direct hyperbilirubinemia, normal or mildly elevated aminotransferases) and examination findings (eg, hepatomegaly, jaundice). Hypercholesterolemia is also common due to reduced cholesterol excretion. Elevated antimitochondrial antibodies are highly characteristic.

Researchers conducted a randomized controlled trial to assess the effectiveness of a new drug to reduce severe cutaneous reactions in patients with rheumatoid arthritis who are treated with adalimumab. Of 150 adults with rheumatoid arthritis who are treated with adalimumab, 75 received the new drug in addition to adalimumab, and 75 received a placebo in addition to adalimumab. Results show that 6 patients in the new drug group developed severe cutaneous reactions, compared to 9 in the placebo group. Which of the following represents the relative risk reduction for severe cutaneous reactions among patients in the new drug group? A. 0.08(9%) B. 0.10(2%) C. 0.12(4%) D. 0.33(61%) E. 0.67(22%) Incorrect

d

A 35-year-old man is found to have elevated alanine aminotransferase and aspartate aminotransferase levels. He has no abdominal pain, nausea, or vomiting. The patient has used intravenous drugs in the past but denies significant alcohol use. His past medical history is negative for blood transfusions or excessive acetaminophen use. Physical examination findings are normal. Laboratory studies show high titers of IgG directed against the hepatitis C envelope protein. Which of the following is the most likely reason that these antibodies do not confer effective immunity against the infection? A.Envelope proteins are lost after recurrent viral replication B.Envelope proteins are sequestered within hepatocytes C.Envelope proteins have low immunogenicity D.Envelope proteins have variations in their antigenic structure E.The antibodies do not have neutralizing properties

d Hepatitis C virus (HCV) has 6 or more genotypes and multiple subtypes that differ in genomic composition by as much as 30%-35%. This is largely due to the fact that the virion-encoded RNA-dependent RNA polymerase has no proofreading 3' → 5' exonuclease activity, which results in many errors during replication. Although most patients are infected with a single genotype, the high mutation rate leads to the development of distinct quasispecies within infected individuals over time. These variant strains differ primarily at hypervariable genomic regions, such as those found in the sequences coding for its envelope glycoproteins. The continuous generation of novel envelope glycoproteins prevents infected individuals from mounting an effective immune response. As production of host antibodies against a quasispecies commences, that strain dies off and new ones take its place. The tremendous antigenic variety of HCV has significantly slowed efforts to develop an effective vaccine.

A 72-year-old woman is evaluated due to a month of fatigue and shortness of breath on exertion. The patient has also had lower back pain but no fever or cough. She takes valsartan for hypertension. Temperature is 37.2 C (99 F), blood pressure is 130/80 mm Hg, pulse is 84/min, and respirations are 14/min. On physical examination, the conjunctivae are pale. The remainder of the examination is normal. Laboratory results are as follows: Hemoglobin8.2 g/dL Creatinine2.1 mg/dL Calcium11.2 mg/dL Total protein8.5 g/dL Albumin3.7 g/dL X-rays of the lower back demonstrate multiple areas of radiolucency in the L4 and L5 vertebrae. Serum protein electrophoresis reveals a monoclonal gamma globulin spike. Further studies are most likely to show which of the following? PTH, urine calcium, VIT D, PTHrp

decrease, increase, decrease, and PTHrp normal

In an animal experiment, mice proerythroblasts are cultured in 2 different growth media; the first medium is folate deficient, whereas the second (control) is supplemented with folic acid. Both media contain high concentrations of erythropoietin. Over 48 hours, cells in the control media proliferate and differentiate into reticulocytes, whereas in the folate-deficient media, cell proliferation is minimal, with the majority of cells undergoing apoptosis. In another experiment, a substance is added to the folate-deficient media, which prevents apoptosis and permits proliferation of the proerythroblasts. Which of the following is the most likely substance added to the growth medium? A.Cobalamin B. Cytosine C.Glutamine D.Homocysteine E.Thymidine

e Folate derivatives are crucial in the synthesis of nucleic acids (particularly thymine, but also purine bases), conversion of homocystine to methionine, and generation of one-carbon carriers (eg, S-adenosylmethionine) for methylation reactions. All biochemical functions of folate require the reduced tetrahydrofolate form of the vitamin, which is synthesized by dihydrofolate reductase (a rate-limiting step in folate metabolism). The enzyme thymidylate synthase is responsible for converting deoxyuridine monophosphate (dUMP) to deoxythymidine monophosphate (dTMP]). Although most enzymes involved in one-carbon metabolism maintain folate in its active tetrahydrofolate form, thymidylate synthase is unique in that it oxidizes 5,10-methylenetetrahydrofolate to dihydrofolate. This makes de novo thymidine synthesis particularly susceptible to folate-deficient conditions, as tetrahydrofolate must be continuously regenerated by dihydrofolate reductase. In this experiment, inhibition of thymidylate synthase due to low folate conditions increases the ratio of dUMP to dTMP, causing the incorporation of uracil into DNA in place of thymidine. This leads to excessive activation of DNA repair mechanisms, resulting in double-stranded DNA breaks and apoptosis. Thymidine supplementation increases the amount of available thymidine and is capable of inhibiting apoptosis in folate-deficient cells.

A 22-year-old man comes to the office due to erectile dysfunction and lack of sexual desire. Medical history is unremarkable. The patient is in a stable relationship with his fiancé and does not use tobacco, alcohol, or illicit drugs. Height is 188 cm (6 ft 2 in) and weight is 88 kg (194 lb). Examination shows bilateral gynecomastia, sparse facial hair, and small, firm testes. The penis is normal in size and peripheral vision is normal on confrontation. This patient would most benefit from which of the following? A.Aromatase inhibitor B.Gonadotropin C.Phosphodiesterase 5 inhibitor D.Selective estrogen receptor modulator E.Testosterone

e This patient has features of testosterone deficiency (ie, hypogonadism), including erectile dysfunction, low libido, and gynecomastia. His tall stature and small, firm testes are consistent with Klinefelter syndrome (KS), which causes hypogonadism due to hyalinization and fibrosis of the testes. Patients who develop hypogonadism prior to puberty, including patients with KS, often have a eunuchoid habitus, sparse body hair, and high-pitched voice, whereas these features are much less pronounced in those who develop hypogonadism later in life. Primary hypogonadism can be confirmed with low serum testosterone associated with elevated LH. Management of male hypogonadism includes testosterone therapy, which can improve libido and erectile function, increase bone density, and facilitate muscle development.

A 53-year-old man comes to the emergency department due to progressive shortness of breath and nonproductive cough. Medical history is significant for long-standing hypertension and type 2 diabetes mellitus, for which he takes lisinopril and metformin. The patient has no drug allergies. Blood pressure is 160/100 mm Hg, pulse is 110/min, and respirations are 20/min. On physical examination, heart sounds are regular. Lung examination reveals decreased tactile fremitus over the lower right lung along with dullness to percussion. Which of the following is the most likely diagnosis? A.Bronchospasm B.Emphysema C.Lobar consolidation D.Pericardial effusion E.Pleural effusion F.Pneumothorax G.Pulmonary edema

e go check the table

A research center studying cardiovascular pathology is conducting trials in which experimental rabbits are fed sweet peas containing substances that alter connective tissue synthesis and maturation. The animals are monitored for several weeks prior to euthanasia. Autopsy shows myxomatous degeneration with pooling of proteoglycans in the media layer and an intact intima layer in large arteries. The pathologic findings seen in the experiment are most similar to which of the following conditions? A.Aortic aneurysm B.Atherosclerosis C.Berry aneurysm D.False aneurysm E.Giant cell arteritis F.Malignant hypertension

so the media is the one that has proteoglycans in it and the intima is fine and because the intima is intact we can cancel out falase aneurysm. recall in atherosclerosis shits happening to intima so thats out to because intima is fine here, its the media that we are concerned with, giant cell arteritis has granulomatous inflammation and is Autoantibody mediated so thats out too. You know that malignant htn causes arteriosclerosis and onion skinning so thats out. Berry aneurysm is related to stress and htn and PKD so thats out we left with aortic aneurysm. now recall that people with marfan have cystic medial degeneration and that shit literally involves the media of the vessel and leads to aneurysm. Myxomatous changes in the media of large arteries is found in cystic medial degeneration, a form of pathologic weakening of connective tissue that occurs in large arteries. Medial degeneration is characterized by the fragmentation of elastic tissue and separation of the elastic and fibromuscular components of the tunica media by small, clefted spaces that become filled with amorphous extracellular matrix (ie, mucopolysaccharide). Marfan syndrome is a frequent cause of cystic medial degeneration in younger patients. It is characterized by an autosomal dominant defect in the extracellular glycoprotein fibrillin-1, which is a major component of the extracellular matrix microfibrils that form the scaffolding for elastic fibers. Mutation of the fibrillin-1 gene results in elastic tissue fragmentation, which predisposes affected patients to aortic aneurysms (with an intact intima) and dissections (with an intimal tear). Beta-aminopropionitrile (a chemical found in certain kinds of sweet peas) causes inhibition of lysyl oxidase, an enzyme responsible for cross-linking elastin fibers and collagen fibers. Ingestion of this compound can cause connective tissue disruption in the aorta that mimics the myxomatous degeneration seen in patients with Marfan syndrom


Conjuntos de estudio relacionados

other 50/100 history final questions (R)

View Set

CRJ 203: Exam 2 Study Guide OFFICIAL

View Set

Into Business Chapter 1-16 Review

View Set

Chapter 3: Human Resource Management

View Set

Chapter 25: Drug Therapy for Seizures

View Set

Chapter 18 - Gravitational Fields

View Set

Protein Energy Malnutrition (PEM)

View Set

1.4 true or false + notes (Phil 120)

View Set

chapter 12 Internet Applications it fundamentals

View Set